HESI Exit Practice Questions and Rationale (2)

¡Supera tus tareas y exámenes ahora con Quizwiz!

A client is diagnosed with a full-thickness burn. What should the nurse anticipate will be used for final coverage of the client's burn wound? 1.Biobrane 2.Autograft 3.Homograft 4.Xenograft

2 A full-thickness burn will require terminal coverage with an autograft-the client's own skin. Biobrane is porcine collagen bonded to a silicone membrane, which is temporary and lasts anywhere from 10 to 21 days. Homografts (cadaveric skin) and xenografts (pigskin) provide temporary coverage of the wound by acting as a dressing for up to 3 weeks before rejecting.

The nurse is caring for a client who has been taking a sulfonamide and should monitor for signs and symptoms of which adverse effects of the medication? Select all that apply. 1.Ototoxicity 2.Palpitations 3.Nephrotoxicity 4.Bone marrow suppression 5.Gastrointestinal (GI) effects 6.Increased white blood cell (WBC) count

3, 4, 5 Adverse effects of sulfonamides include nephrotoxicity, bone marrow suppression, GI effects, hepatotoxicity, dermatological effects, and some neurological symptoms, including headache, dizziness, vertigo, ataxia, depression, and seizures. Options 1, 2, and 6 are unrelated to these medications.

The nurse is caring for a client with a diagnosis of Chlamydia. Because the client has a history of noncompliance with medication administration, the health care provider prescribes azithromycin. When educating the client about azithromycin, the nurse should make which statement? 1."One dose of this medication will be needed." 2."This medication is given only every 72 hours." 3."You will need to take this medication every 6 hours." 4."You will need to return tomorrow for your second dose."

1 Azithromycin is a macrolide antibiotic. The usual pharmacological treatment for urethral, cervical, or rectal chlamydial infections is doxycycline or azithromycin. Azithromycin is often prescribed when compliance may be a problem because only one dose is needed; however, expense is a concern with this medication. The responses in the remaining options are incorrect.

The home health nurse is visiting a client who has been started on therapy with clotrimazole. The nurse should monitor which item to monitor the effectiveness of this medication with each visit? 1.Rash 2.Fever 3.Pain relief 4.Sore throat

1 Clotrimazole is a topical antifungal agent used in the treatment of cutaneous fungal infections. The nurse monitors the effectiveness of this medication by noting the presence or absence of a skin rash, which is characteristic of this infection. Fever, pain relief, and sore throat are unrelated to the use of this medication.

The nurse has provided medication instructions to a client with an iron deficiency anemia who will be taking iron supplements. Which statement made by the client indicates an understanding of this medication? 1."I need to increase my fluid intake." 2."I should eliminate fiber foods from my diet." 3."I need to take the medication with water before a meal." 4."I should be sure to chew the tablet thoroughly before swallowing it."

1 Iron preparations can be very irritating to the stomach and are best taken between meals. Because iron supplements may be associated with constipation, the client should increase fluids and fiber in the diet to counteract this side effect of therapy. Iron preparations should be taken with a substance that is high in vitamin C to increase its absorption. The tablet is swallowed whole and not chewed.

A client has a prescription for ketoconazole. Which instruction should the nurse include in the client's teaching plan? 1.Avoid exposure to sunlight. 2.Take the medication with an antacid. 3.Take the medication on an empty stomach. 4.Limit alcohol consumption to 2 ounces per day.

1 Ketaconazole is an antifungal medication. The client also should avoid exposure to sunlight because the medication increases photosensitivity. Antacids should be avoided for 2 hours after ketoconazole is taken because gastric acid is needed to activate the medication; however, it should be taken with food. The client should avoid concurrent use of alcohol because the medication is hepatotoxic.

A nursing instructor is reviewing information on the organs of the immune system. The instructor asks a nursing student to name the location of Kupffer cells. Which organ identified by the nursing student indicates successful teaching? 1.The liver 2.The spleen 3.The tonsils 4.Bone marrow

1 The liver contains a large number of macrophages called Kupffer cells. Kupffer cells are a part of the body's reticuloendothelial system and is a protective function of the liver.They help filter blood by phagocytizing microorganisms and other foreign particles passing through the liver. The organs in the remaining options are incorrect.

The community health nurse is conducting a research study and is identifying clients in the community at risk for latex allergy. Which client population is most at risk for developing this type of allergy? 1.Hairdressers 2.The homeless 3.Children in day care centers 4.Individuals living in a group home

1 Individuals most at risk for developing a latex allergy include health care workers; individuals who work in the rubber industry; or those who have had multiple surgeries, have spina bifida, wear gloves frequently (such as food handlers, hairdressers, and auto mechanics), or are allergic to kiwis, bananas, pineapples, tropical fruits, grapes, avocados, potatoes, hazelnuts, or water chestnuts.

A pediatric nurse has obtained ribavirin in powder form from the pharmacy to administer to a child with respiratory syncytial virus (RSV) infection. After preparing the medication, the nurse should administer it by which route? 1.Inhalation 2.Intravenous 3.Subcutaneous 4.Oral, in the child's formula

1 Ribavirin is an antiviral medication and is active against RSV, influenza virus types A and B, and herpes simplex virus. It is administered by oral inhalation. The medication is absorbed from the lungs and achieves high concentrations in respiratory tract secretions and erythrocytes. It is not administered by the routes identified in the remaining options.

Which interventions apply in the care of a client at high risk for an allergic response to a latex allergy? Select all that apply. 1.Use nonlatex gloves. 2.Use medications from glass ampules. 3.Place the client in a private room only. 4.Keep a latex-safe supply cart available in the client's area. 5.Avoid the use of medication vials that have rubber stoppers. 6.Use a blood pressure cuff from an electronic device only to measure the blood pressure.

1, 2, 4, 5 If a client is allergic to latex and is at high risk for an allergic response, the nurse would use nonlatex gloves and latex-safe supplies, and would keep a latex-safe supply cart available in the client's area. Any supplies or materials that contain latex would be avoided. These include blood pressure cuffs and medication vials with rubber stoppers that require puncture with a needle. It is not necessary to place the client in a private room.

The charge nurse is planning the assignment for the day. Which factors should the nurse remain mindful of when planning the assignment? Select all that apply. 1.The acuity level of the clients 2.Specific requests from the staff 3.The clustering of the rooms on the unit 4.The number of anticipated client discharges 5.Client needs and workers' needs and abilities

1, 5 There are guidelines that the nurse should use when delegating and planning assignments. These include the following: ensure client safety; be aware of individual variations in work abilities; determine which tasks can be delegated and to whom; match the task to the delegatee on the basis of the nurse practice act and appropriate position descriptions; provide directions that are clear, concise, accurate, and complete; validate the delegatee's understanding of the directions; communicate a feeling of confidence to the delegatee and provide feedback promptly after the task is performed; and maintain continuity of care as much as possible when assigning client care. Staff requests, convenience as in clustering client rooms, and anticipated changes in unit census are not specific guidelines to use when delegating and planning assignments.

The client with human immunodeficiency virus (HIV) infection has been started on therapy with zidovudine. The nurse reviews the laboratory results and determines that the client is experiencing an adverse effect of the medication if which is noted? 1.Phosphorus 4.5 mg/dL (1.45 mmol/L) 2.Hemoglobin of 10 g/dL (100 mmol/L) 3.Blood glucose level 70 mg/dL (4 mmol/L) 4.Blood urea nitrogen (BUN) 10 mg/dL (3.6 mmol/L)

2 An adverse effect of this medication therapy is agranulocytopenia and anemia. The nurse carefully monitors the CBC count for these changes. With early HIV infection or in the client who is asymptomatic, CBC counts are monitored monthly for 3 months and then every 3 months thereafter. In clients with advanced disease, these counts are monitored every 2 weeks for the first 2 months and then once a month if the medication is tolerated well. The normal hemoglobin level is 14 to18 g/dL (140 to180 mmol/L); thus this client is experiencing anemia. The remaining options identify normal values. The normal phosphorus 3.0 to4.5 mg/dL (0.97 to 1.45 mmol/L). The normal blood glucose level is 70 to110 mg/dL (4 to 6 mmol/L). The normal BUN is 10 to 20 mg/dL (3.6 to 7.1 mmol/L).

The nurse is planning the client assignments for the shift. Which client should the nurse assign to the unlicensed assistive personnel (UAP)? 1.A client requiring dressing changes 2.A client requiring frequent temperature measurements 3.A client on a bowel management program requiring rectal suppositories and a daily enema 4.A client with diabetes mellitus requiring daily insulin and reinforcement of dietary measures

2 Assignment of tasks to the UAP needs to be made based on job description, level of clinical competence, and state law. Options 1, 3, and 4 involve care that requires the skill of a licensed nurse. The client described in the correct option has needs that can be met by a UAP.

A client has requested and undergone testing for human immunodeficiency virus (HIV) infection. The client asks what will be done next because the result of the enzyme-linked immunosorbent assay (ELISA) has been positive. Which diagnostic study should the nurse be aware of before responding to the client? 1.No further diagnostic studies are needed. 2.A Western blot will be done to confirm these findings. 3.The client probably will have a bone marrow biopsy done. 4.A CD4+ cell count will be done to measure T helper lymphocytes.

2 Human immunodeficiency virus (HIV) can cause acquired immunodeficiency syndrome, which is a viral disease that destroys T cells, thereby increasing susceptibility to infection and malignancy. If the result of the ELISA is positive, the Western blot is done to confirm the findings. If the result of the Western blot is positive, the client is considered to be seropositive for the infection and to be infected with the virus. The remaining options are incorrect.

A client presents at the health care provider's office with complaints of a bulls-eye rash on his upper leg. Which question should the nurse ask first? 1."Do you have any cats in your home?" 2."Have you been camping in the last month?" 3."Have you or close contacts had any flu-like symptoms within the last few weeks?" 4."Have you been in physical contact with anyone who has the same type of rash?"

2 The nurse should ask questions to assist in identifying the cause of Lyme disease, which is a multisystem infection that results from a bite by a tick carried by several species of deer. The rash from a tick bite can be a ring-like rash occurring 3 to 4 weeks after a bite and is commonly seen on the groin, buttocks, axillae, trunk, and upper arms or legs. Option 1 is referring to toxoplasmosis, which is caused by the inhalation of cysts from contaminated cat feces. Lyme disease cannot be transmitted from one person to another.

A pregnant client who has human immunodeficiency virus (HIV) infection is being seen in the antenatal clinic. The nurse expects the health care provider (HCP) to initiate zidovudine at how many weeks of gestation? 1.4 2.14 3.24 4.34

2 The pregnant woman with HIV infection will be prescribed oral zidovudine in the 14th week of gestation. Before this time, the fetus is at risk because of the teratogenic effects of the medication. In addition, a bolus of zidovudine is given intravenously during labor, and the neonate is treated for 6 weeks after birth

A nurse provides dietary instructions to a client who will be taking warfarin sodium. The nurse should tell the client to avoid which food item? 1.Grapes 2.Spinach 3.Watermelon 4.Cottage cheese

2 Warfarin sodium is an anticoagulant. Anticoagulant medications act by antagonizing the action of phytonadione, which is needed for clotting. When a client is taking an anticoagulant, foods high in phytonadione often are omitted from the diet. Phytonadione-rich foods include green leafy vegetables, fish, liver, coffee, and tea.

The registered nurse (RN) is planning assignments for the clients on a nursing unit. The RN needs to assign 4 clients and has 1 RN, 1 licensed practical (vocational) nurse, and 2 unlicensed assistive personnel (UAPs) on a nursing team. Which client would the nurse most appropriately assign to the licensed practical nurse? 1.The client who requires a 24-hour urine collection 2.The client with an abdominal wound requiring frequent wound irrigations 3.The older client requiring assistance with a bed bath and frequent ambulation 4.The client on a mechanical ventilator requiring frequent assessment and suctioning

2 When delegating nursing assignments, the nurse must consider the skills and educational level of the nursing staff. The licensed practical (vocational) nurse is skilled in wound irrigation and dressing changes, so this client would be assigned to this staff member. Collecting 24-hour urine and helping with a bed bath and frequent ambulation can most appropriately be assigned to the UAPs. The client on the mechanical ventilator requiring frequent assessment and suctioning should most appropriately be cared for by the RN.

A client is receiving amoxicillin orally every 8 hours. Which finding would indicate to the nurse that the client is experiencing a frequent minor side effect related to the medication? 1.Fever 2.Vaginal drainage 3.Severe watery diarrhea 4.Severe abdominal cramps

2 Amoxicillin is a type of penicillin. Frequent minor side effects include gastrointestinal disturbances, headache, and oral or vaginal candidiasis (perineal itching). A less common but more harmful effect that can occur include superinfection, such as potentially fatal antibiotic-associated colitis, which results from altered bacterial balance. Symptoms and signs include abdominal cramps, severe watery diarrhea, and fever.

The nurse caring for a client who is taking an aminoglycoside should monitor the client for which adverse effects of the medication? Select all that apply. 1.Seizures 2.Ototoxicity 3.Renal toxicity 4.Dysrhythmias 5.Hepatotoxicity

2, 3, 4 Aminoglycosides are administered to inhibit the growth of bacteria. Adverse effects of this medication include confusion, ototoxicity, renal toxicity, gastrointestinal irritation, palpitations (dysrhythmias), blood pressure changes, and hypersensitivity reactions. Therefore, the remaining options are incorrect.

The nurse is creating a plan of care for a client who is receiving amphotericin B intravenously.A main component of the plan of care is monitoring for adverse effects related to the administration of this medication. Which should the nurse include in a list of manifestations to watch for? 1.Fatigue 2.Confusion 3.Visual difficulties 4.Increased urinary output

3 Amphotericin B is an antifungal. Vision and hearing alterations, seizures, hepatic failure, paresthesias (tingling, numbness, or pain in the hands and feet), and coagulation defects also occur. Other adverse effects include nephrotoxicity, which occurs commonly and is evidenced by decreased urine output. Cardiovascular toxicity (as evidenced by hypotension and ventricular fibrillation) and anaphylactic reaction occur rarely.

Azathioprine is prescribed for a client to suppress rejection of a renal transplant. In planning for administration of the medication, the nurse understands that which description is the mechanism of action of this medication? 1.It crosslinks DNA. 2.It blocks all T cell functions. 3.It inhibits the proliferation of B and T lymphocytes. 4.It decreases the activity of thymus-derived lymphocytes.

3 Azathioprine is an immunosuppressant; it suppresses cell-mediated and humoral immune responses by inhibiting the proliferation of B and T lymphocytes. It generally is used as an adjunct to cyclosporine and glucocorticoids to help suppress transplant rejection. The remaining options are incorrect mechanisms of action.

The nurse is caring for a postrenal transplantation client taking cyclosporine. The nurse notes an increase in one of the client's vital signs, and the client is complaining of a headache. What vital sign is most likely increased? 1.Pulse 2.Respirations 3.Blood pressure 4.Pulse oximetry

3 Cyclosporine is an immunosuppressant used for preventionof rejection following allogeneic organ transplantation. Hypertension can occur in a client taking cyclosporine and because this client is also complaining of a headache, the blood pressure is the vital sign to be monitored most closely. Other adverse effects include infection, nephrotoxicity, and hirsutism. The remaining options are unrelated to the use of this medication.

A client with chronic kidney disease is receiving epoetin alfa for the past 2 months. What should the nurse determine is an indicator that this therapy is effective? 1.A decrease in blood pressure 2.An increase in white blood cells 3.An increase in serum hematocrit 4.A decrease in serum creatinine level

3 Epoetin alfa stimulates red blood cell production. Initial effects should be seen within 1 to 2 weeks, and the hematocrit reaches normal levels in 2 to 3 months.

A client arrives at the health care clinic and tells the nurse that he was just bitten by a tick and would like to be tested for Lyme disease. The client reports that he removed the tick and flushed it down the toilet. The nurse should take which nursing action? 1.Refer the client for a blood test immediately. 2.Ask the client about the size and color of the tick. 3.Tell the client to return to the clinic in 4 to 6 weeks. 4.Inform the client that the tick is needed to perform a test.

3 Lyme disease is a reportable systemic infectious disease caused by the spirochete Borellia burgdorferi and results from the bite of an infected deer tick, also known as the black-legged tick. A blood test is available to detect Lyme disease; however, it is not a reliable test if performed before 4 to 6 weeks after the tick bite. Antibody formation takes place in the following manner: Immunoglobulin M (IgM) is detected 3 to 4 weeks after Lyme disease onset, peaks at 6 to 8 weeks, then gradually disappears; IgG is detected 2 to 3 months after infection and may remain elevated for years. The actions in the remaining options are inaccurate.

A client admitted to the hospital is taking zidovudine. The nurse monitors the client for which adverse effect of the medication? 1.Colitis 2.Ototoxicity 3.Neurotoxicity 4.Visual disturbances

3 Human immunodeficiency virus (HIV) can cause acquired immunodeficiency syndrome, which is a viral disease that destroys T cells, thereby increasing susceptibility to infection and malignancy. Zidovudine is a nucleoside reverse transcriptase inhibitor that is used in combination with other antiretroviral agents to treat human immunodeficiency virus (HIV) infections. Adverse effects include anemia, granulocytopenia, and neurotoxicity as evidenced by ataxia, fatigue, lethargy, and nystagmus. Seizures can also occur. Colitis, ototoxicity, and visual disturbances are not adverse effects of this medication.

The nurse is preparing to administer pentamidine to an assigned client by the intravenous route. The nurse plans to monitor which item most closely after administering this medication? 1.Capillary refill 2.Peripheral pulses 3.Blood pressure (BP) 4.Level of consciousness

3 Pentamidine is an antiinfective medication. Life-threatening and fatal hypotension can occur after the administration of pentamidine. The client must be in a supine position with frequent BP checks after administration. The remaining options are not associated with the administration of this medication.

A nurse needs to maintain food and fluid intake to minimize the risk of dehydration in a client with diabetes mellitus who has gastroenteritis. Which is the appropriate nursing intervention? 1.Offer water only until the client is able to tolerate solid foods. 2.Withhold all fluids until vomiting has ceased for at least 4 hours. 3.Encourage the client to take 8 to 12 oz of fluid every hour while awake. 4.Maintain a clear liquid diet for at least 5 days before advancing to solids.

3 Small amounts of fluid may be tolerated, even when vomiting is present. The nurse should encourage liquids containing glucose and electrolytes every hour. The remaining options will not provide the adequate intake needed by the client with diabetes mellitus.

The nurse is reviewing the laboratory test results for a client with a diagnosis of Cushing's syndrome. Which laboratory finding would the nurse expect to note in this client? 1.A platelet count of 200,000 mm3 (200 × 109/L) 2.A blood glucose level of 110 mg/dL (6.28 mmol/L) 3.A potassium (K+) level of 3.0 mEq/L (3.0 mmol/L) 4.A white blood cell (WBC) count of 6000 mm3 (6 × 109/L)

3 The client with Cushing's syndrome experiences hypokalemia, hyperglycemia, an elevated WBC count, and elevated plasma cortisol and adrenocorticotropic hormone levels. These abnormalities are caused by the effects of excess glucocorticoids and mineralocorticoids in the body. The laboratory values listed in the remaining options would not be noted in the client with Cushing's syndrome.

The nurse is preparing to care for a client with immunodeficiency. The nurse should plan to address which problem as the priority? 1.Anxiety 2.Fatigue 3.Risk for infection 4.Need for social isolation

3 The client with immunodeficiency has inadequate or no immune bodies and is at risk for infection. The priority concern would be risk for infection. The question presents no data indicating that the client is experiencing anxiety. Fatigue may be a problem and the client may need to be placed on protective isolation, but these are not the priority problems for this client. Infection can be life-threatening and is the priority.

The nurse is preparing a client with a new diagnosis of hypothyroidism for discharge. The nurse determines that the client understands discharge instructions if the client states that which signs and symptoms are associated with this diagnosis? Select all that apply. 1.Tremors 2.Weight loss 3.Feeling cold 4.Loss of body hair 5.Persistent lethargy 6.Puffiness of the face

3, 4, 5, 6 Feeling cold, hair loss, lethargy, and facial puffiness are signs of hypothyroidism. Tremors and weight loss are signs of hyperthyroidism.

The nurse manager is planning the clinical assignments for the day. Which staff members cannot be assigned to care for a client with herpes zoster? Select all that apply. 1.The nurse who never had roseola 2.The nurse who never had mumps 3.The nurse who never had chickenpox 4.The nurse who never had German measles 5.The nurse who never received the varicella-zoster vaccine

3, 5 The nurses who have not had chickenpox or did not receive the varicella-zoster vaccine are susceptible to the herpes zoster virus and should not be assigned to care for the client with herpes zoster. Nurses who have not contracted roseola, mumps, or rubella are not necessarily susceptible to herpes zoster. Herpes zoster (shingles) is caused by a reactivation of the varicella-zoster virus, the causative virus of chickenpox. Individuals who have not been exposed to the varicella-zoster virus or who did not receive the varicella-zoster vaccine are susceptible to chickenpox. Health care workers who are unsure of their immune status should have varicella titers done before exposure to a person with herpes zoster.

The nurse witnessed a vehicle hit a pedestrian. The victim is dazed and tries to get up. A leg appears fractured. Which intervention should the nurse take? 1.Try to reduce the fracture manually. 2.Assist the victim to get up and walk to the sidewalk. 3.Leave the victim for a few moments to call an ambulance. 4.Stay with the victim and encourage him or her to remain still.

4 With a suspected fracture, the victim is not moved unless it is dangerous to remain in that spot. The nurse should remain with the victim and have someone else call for emergency help. A fracture is not reduced at the scene. Before the victim is moved, the site of fracture is immobilized to prevent further injury.

A client with human immunodeficiency virus infection is taking indinavir. The nurse plans to provide the client with which direction when providing instructions about the use of this medication? 1.Store the medication in the refrigerator. 2.Take the medication with a high-fat snack. 3.Take the medication with the large meal of the day. 4.Take the medication with water on an empty stomach.

4 Acquired immunodeficiency syndrome is a viral disease caused by the human immunodeficiency virus (HIV), which destroys T cells, thereby increasing susceptibility to infection and malignancy. Indinavir is a protease inhibitor. To maximize absorption, the medication should be administered with water on an empty stomach. The medication may be taken 1 hour before a meal or 2 hours after a meal, or it may be administered with skim milk, coffee, tea, or a low-fat meal. It is not administered with a large meal. The medication should be stored at room temperature and protected from moisture because moisture can degrade the medication.

A client with acquired immunodeficiency syndrome (AIDS) is experiencing nausea and vomiting. The nurse should include which measure in the dietary plan? 1.Provide large, nutritious meals. 2.Serve foods while they are hot. 3.Add spices to food for added flavor. 4.Remove dairy products and red meat from the meal.

4 Acquired immunodeficiency syndrome is a viral disease caused by the human immunodeficiency virus (HIV), which destroys T cells, thereby increasing susceptibility to infection and malignancy. The client with AIDS who has nausea and vomiting should avoid fatty products such as dairy products and red meat. Meals should be small and frequent to lessen the chance of vomiting. The client should avoid spices and odorous foods because they aggravate nausea. Foods are best tolerated cold or at room temperature.

The nurse is monitoring a client with herpes simplex virus (HSV) who is receiving intravenous (IV) acyclovir. Which laboratory result would be of concern as a possible adverse effect of this medication? 1.Platelet count of 300,000 mm3 (300 × 109/L) 2.Prothrombin time of 12 seconds (12 seconds) 3.White blood cell count of 6000 mm3 (6 × 109/L) 4.Blood urea nitrogen (BUN) of 36 mg/dL (12.9 mmol/L)

4 Acyclovir is an antiviral medication. Although the most common side and adverse reactions with this medication are phlebitis and inflammation at the IV site, reversible nephrotoxicity, evidenced by elevated serum creatinine and BUN levels, can occur in some clients. The cause of nephrotoxicity is deposition of acyclovir in the renal tubules. The risk of renal injury is increased by dehydration and the use of other nephrotoxic medications. The laboratory values identified in the remaining options are within normal limits.

The nurse is providing instructions to the parent of a child with iron deficiency anemia about the administration of a liquid oral iron supplement. Which statement, if made by the parent, indicates an understanding of the administration of this medication? 1. "I should give the iron with food." 2."I can mix the iron with cereal to give it." 3."I should add the iron to the formula in the baby's bottle." 4."I should use a medicine dropper and place the iron near the back of the throat."

4 An oral iron supplement should be administered through a straw or medicine dropper placed at the back of the mouth because it will stain the teeth. The parents should be instructed to brush or wipe the teeth after administration. Iron is administered between meals because absorption is decreased if there is food in the stomach. Iron requires an acidic environment to facilitate its absorption in the duodenum.

The nurse is caring for a postpartum client with a diagnosis of deep vein thrombosis who is receiving a continuous intravenous infusion of heparin sodium. Review of which laboratory result is the most important by the nurse? 1.Platelet count 2.Prothrombin time (PT) 3.International normalized ratio (INR) 4.Activated partial thromboplastin time (aPTT)

4 Anticoagulation therapy may be used to prevent the extension of thrombus by delaying the clotting time of the blood. The aPTT time should be monitored, and the heparin sodium dose should be adjusted to maintain a therapeutic level of 1.5 to 2.5 times the control value in seconds. The platelet count cannot be used to determine an adequate dosage for the heparin sodium infusion. The PT and the INR are used to monitor coagulation time when warfarin sodium is used.

The nurse has given the client with atrial fibrillation instructions to take one 1 aspirin daily. The client says to the nurse, "Why do I need to take this? I don't get any pain with my heart rhythm."Which response by the nurse is the most appropriate? 1."This will keep you from experiencing chest pain." 2."This will most likely keep you from ever having a heart attack." 3."This will prevent any inflammation from occurring on the walls of your heart." 4."This will help prevent clot formation in your heart as a result of your heart's rhythm."

4 Atrial fibrillation puts the client at risk for mural thrombi because of the sluggish blood flow through the atria that occurs as a result of loss of the atrial kick. In atrial fibrillation, the health care provider may prescribe a daily aspirin. This will prevent clot formation along the walls of the atria and resultant embolus. Aspirin will not prevent chest pain or keep a client from ever having a heart attack. Although aspirin does have antiinflammatory properties, it cannot prevent any inflammation from occurring, as stated in option 3.

The nursing student enrolled in an anatomy and physiology course is studying the immune system. The nursing instructor determines that the student understands the chemical barriers against a nonspecific immune response if which statement is made? 1."The skin is considered a chemical barrier." 2."The mucous membranes act as chemical barriers." 3."The cilia lining the respiratory tract are chemical barriers." 4."Acids and enzymes found in body fluids function as chemical barriers."

4 Chemical barriers include various acids and enzymes found in body fluids. The skin, the mucous membranes, and the action of cilia lining the respiratory tract are physical barriers.

The nurse is performing an assessment on a client with a diagnosis of pernicious anemia. Which finding would the nurse expect to note in this client? 1.Dyspnea 2.Dusky mucous membranes 3.Shortness of breath on exertion 4.Red tongue that is smooth and sore

4 Classic signs of pernicious anemia include weakness, mild diarrhea, and a smooth red tongue that is sore. The client also may have nervous system signs and symptoms such as paresthesias, difficulty with balance, and occasional confusion. The client does not exhibit dyspnea, the mucous membranes do not become dusky, and the client does not exhibit shortness of breath.

The home care nurse is making a monthly visit to a client with a diagnosis of pernicious anemia who has been receiving a monthly injection of cyanocobalamin. Before administering the injection, the nurse evaluates the effects of the medication and determines that a therapeutic effect is occurring if the client makes which statement? 1."I feel really lightheaded." 2."I no longer have any nausea." 3."I have not had any pain in a month." 4."I feel stronger and have a much better appetite."

4 Cyanocobalamin is essential for DNA synthesis. It can take up to 3 years for the vitamin B12 stores to be depleted and for symptoms of pernicious anemia to appear. Symptoms can include weakness, fatigue, anorexia, loss of taste, and diarrhea. To correct deficiencies, a crystalline form of vitamin B12, cyanocobalamin, can be given intramuscularly. The client statements in options 1, 2, and 3 do not identify a therapeutic effect of the medication.

The client in chronic kidney disease is receiving epoetin alfa. The nurse should monitor this client for which side/adverse effect of this medication? 1.Fever 2.Depression 3.Bradycardia 4.Hypertension

4 Epoetin alfa is generally well tolerated, although hypertension can occur and is the most significant adverse effect. Occasionally, tachycardia may also occur. It may also cause an improved sense of well-being. Fever, depression, and bradycardia are not adverse effects of this medication.

The nurse is collecting subjective and objective data from a client and notes that the client is taking abacavir. The nurse determines that this medication has been prescribed to treat which condition? 1.Otitis media 2.Heart failure 3.Urinary tract infection 4.Human immunodeficiency virus (HIV) infection

4 Human immunodeficiency virus (HIV) can cause acquired immunodeficiency syndrome, which is a viral disease that destroys T cells, thereby increasing susceptibility to infection and malignancy. Abacavir is an antiviral medication and is used to treat HIV infection, in combination with other agents. It is not used to treat the conditions noted in the other options.

A nursing graduate is attending an agency orientation regarding the nursing model of practice implemented in the health care facility. The nurse is told that the nursing model is a team nursing approach. The nurse determines that which scenario is characteristic of the team-based model of nursing practice? 1.Each staff member is assigned a specific task for a group of clients. 2.A staff member is assigned to determine the client's needs at home and begin discharge planning. 3.A single registered nurse (RN) is responsible for providing care to a group of 6 clients with the aid of an unlicensed assistive personnel (UAP). 4.An RN leads 2 licensed practical nurses (LPNs) and 3 UAPs in providing care to a group of 12 clients.

4 In team nursing, nursing personnel are led by a registered nurse leader in providing care to a group of clients. Option 1 identifies functional nursing. Option 2 identifies a component of case management. Option 3 identifies primary nursing (relationship-based practice).

The nurse has received her client assignment for the day. Which client should the nurse care for first? 1.A client requiring a preoperative intravenous antibiotic 2.A client with emphysema who has shortness of breath after just ambulating 3.A client with serous drainage on an incisional spinal wound post laminectomy 4.A client with postoperative pain reported at 7 out of 10, with 10 being the worst

4 In this situation, the client with the pain reported at 7 out of 10 should be cared for first. The pain will intensify and be harder to manage if treatment is delayed. Caring for the client in pain may delay administration of the preoperative antibiotic but does not jeopardize safe and effective care. Shortness of breath is expected in a client with emphysema after ambulation and therefore is not the priority. Serous drainage is expected from a surgical incision and does not indicate an emergency.

A client is prescribed a liquid iron preparation that has the potential to stain the teeth. The nurse should instruct the client to take which action to prevent staining of the teeth? 1.Brush the teeth before drinking the iron. 2.Drink the iron undiluted for maximal effect. 3.Dilute more than the amount prescribed to obtain the correct dosage. 4.Dilute the iron in juice, drink it through a straw, and rinse the mouth afterward.

4 Liquid iron preparations will stain the teeth. The best advice for the client who needs liquid iron is to dilute the iron in juice or water, drink it through a straw, and rinse the mouth well afterward. Brushing before taking the liquid iron would not be of any benefit. The nurse would not instruct a client to take more than the prescribed amount.

Assessment and diagnostic evaluation reveal that a client seen in the ambulatory care clinic has stage II Lyme disease. The clinic nurse identifies which assessment finding as most characteristic of this stage? 1.Arthralgias 2.Joint enlargement 3.Erythematous rash 4.Cardiac conduction deficits

4 Lyme disease is a reportable systemic infectious disease caused by the spirochete Borellia burgdorferi and results from the bite of an infected deer tick, also known as the black-legged tick. Stage II of Lyme disease develops within 1 to 6 months in a majority of untreated persons. The most serious problems include cardiac conduction deficits and neurological disorders such as Bell's palsy and paralysis. Arthralgias and joint enlargement are noted in stage III. A rash appears in stage I.

The nurse is providing dietary instructions to the client with anemia. The client tells the nurse that the iron pills are very expensive and it will be difficult to pay for the pills and buy the proper food. What is the most appropriate nursing response? 1."You will have to find a way to afford both." 2."You will be fine as long as you take the iron pills." 3."Why don't you ask your family to help you out financially?" 4."Would you like for me to check into some other options for you?"

4 Option 4 is correct because it validates the client's issue with cost. The nurse offers help in a nonthreatening manner that will allow the client to accept or decline. Option 2 is incorrect because the client needs to consume a proper diet. Options 1 and 3 block the communication process and are nontherapeutic and nonhelpful statements.

A client is suspected of having systemic lupus erythematosus (SLE). On reviewing the client's record, the nurse should expect to note documentation of which characteristic sign of SLE? 1.Fever 2.Fatigue 3.Skin lesions 4.Elevated red blood cell count

4 Systemic lupus erythematosus is a chronic, progressive, inflammatory connective tissue disorder that can cause major body organs and systems to fail. The major skin manifestation of SLE is a dry, scaly, raised rash on the face known as the butterfly rash. Fever and fatigue may occur before and during exacerbation, but these signs and symptoms are vague. Anemia is most likely to occur in SLE.

A client who is scheduled to have warfarin sodium therapy has a prothrombin time (PT) of 28 seconds (28 seconds). What is the most appropriate nursing intervention at this time? 1.Give double the dose. 2.Administer the next dose. 3.Give half of the next dose. 4.Call the health care provider (HCP).

4 The PT is one test that may be used to monitor warfarin sodium therapy. The international normalized ratio is another laboratory test used to monitor warfarin therapy. The normal PT is 11 to 12.5 seconds (11 to 12.5 seconds). A PT of 28 seconds represents an elevated value. The nurse should withhold the next dose and notify the HCP. A medication dose should not be changed without a specific prescription (options 1 and 3).

The nurse has provided instructions to a client with sickle cell disease regarding measures that will prevent a sickle cell crisis. Which client statement indicates an understanding of these measures? 1."I need to avoid any exercise." 2."I need to avoid increasing my fluid intake." 3."I need to avoid going outdoors in warm weather." 4."I need to avoid situations that may lead to an infection."

4 The client should avoid infections, which can increase metabolic demands and cause dehydration, precipitating a sickle cell crisis. Fluids are important to prevent dehydration, which could lead to sickle cell crisis. Warm weather and mild exercise do not need to be avoided, but the client should take measures to avoid dehydration during these conditions.

The registered nurse is creating the plan for client assignments for the day. Which is the most appropriate assignment for the unlicensed assistive personnel (UAP)? 1.A client scheduled to receive a blood transfusion 2.A client with bladder cancer who will be receiving chemotherapy 3.A client newly diagnosed with diabetes mellitus scheduled for discharge 4.A client on bed rest who requires range-of-motion (ROM) exercises every 4 hours

4 The nurse must determine the most appropriate assignment based on the skills of the staff member and the needs of the client. In this case, the most appropriate assignment for the UAP would be to care for the client on bed rest who requires ROM exercises. The UAP is trained in this procedure. The client receiving chemotherapy and the client receiving a blood transfusion require assessment skills that only a licensed nurse can perform. The client with diabetes mellitus who is being discharged will require predischarge review of diabetic management instructions and coordination of necessary home care services.

The nurse is planning the client assignments for a group of clients and has a licensed practical nurse (LPN) and an unlicensed assistive personnel (UAP) on the nursing team. Which client would the nurse most appropriately assign to the LPN? 1.A client with stable heart failure who has early-stage Alzheimer's disease 2.A client who is scheduled for an electrocardiogram and a chest x-ray examination 3.A client who was treated for dehydration, is weak, and needs assistance with bathing 4.A client with emphysema who is receiving oxygen at 2 L/min by nasal cannula and becomes dyspneic on exertion

4 The nurse would most appropriately assign the client with emphysema to the LPN. This client has an airway problem and has the highest priority needs among the clients presented in the options. The clients described in options 1, 2, and 3 can appropriately be cared for by the UAP.

The nurse is assigned to care for a client with cytomegalovirus retinitis and acquired immunodeficiency syndrome (AIDS) who is receiving foscarnet. The nurse should monitor the results of which laboratory study while the client is taking this medication? 1.CD4 cell count 2.Lymphocyte count 3.Serum albumin level 4.Serum creatinine level

4 Acquired immunodeficiency syndrome is a viral disease caused by the human immunodeficiency virus (HIV), which destroys T cells, thereby increasing susceptibility to infection and malignancy. Cytomegalovirus retinitis is an opportunistic viral infection of the eye. Foscarnet is an antiviral medication that is toxic to the kidneys. The serum creatinine level is monitored before therapy, 2 or 3 times per week during induction therapy, and at least weekly during maintenance therapy. Foscarnet also may cause decreased levels of calcium, magnesium, phosphorus, and potassium. Thus, these levels also are measured with the same frequency.

Cyclosporine is prescribed to be administered by the intravenous (IV) route. Which is an inappropriate action in preparing and administering this medication? 1.Mixing the solution and covering it with a paper bag 2.Administering the medication over a period of 2 to 6 hours 3.Mixing 1 mL of concentrate in 50 mL of 0.9% sodium chloride and administering by infusion 4.Mixing 1 mL of concentrate in 10 mL of 0.9% sodium chloride and administering by bolus injection

4 Cyclosporine is an immunosuppressant medication used to prevent rejection following allogeneic organ transplantation. For IV administration of cyclosporine, 1 mL of concentrate is diluted in 20 to 100 mL of 0.9% sodium chloride or 5% dextrose. The solution should be protected from light. The initial dose is 5 to 6 mg/kg (one third of the oral dose) administered over a 2- to 6-hour infusion.

Itraconazole is prescribed for a client to treat a fungal infection. The nurse monitors the client closely for which manifestation that is indicative of an adverse effect? 1.Diarrhea 2.Headache 3.Increased urine output 4.Anorexia and abdominal pain

4 Itraconazole is an antifungal medication. Hepatitis is an adverse effect associated with the medication, and if anorexia of any degree, abdominal pain, unusual tiredness or weakness, dark urine, or jaundice develops, the health care provider should be notified.

A client who was diagnosed with toxic shock syndrome (TSS) now exhibits petechiae, oozing from puncture sites, and coolness of the digits of the hands and feet. Clotting times determined for this client are prolonged. The nurse interprets these clinical signs as being most compatible with which condition? 1.Heparin overdose 2.Vitamin K deficiency 3.Factor VIII deficiency 4.Disseminated intravascular coagulopathy (DIC)

4 TSS is caused by infection and often is associated with tampon use. The client's clinical signs in this question are compatible with DIC, which is a complication of TSS. The nurse assesses the client at risk and notifies the health care provider promptly when signs and symptoms of DIC are noted. Although signs of bleeding may be seen with each of the conditions listed in the incorrect options, the initial diagnosis of TSS makes DIC the logical correct option.

A child has been prescribed tetracycline hydrochloride. The nurse providing medication information to the mother should plan to emphasize which instruction about giving this medication to the child? 1.Give the medication with milk. 2.Give the medication with ice cream. 3.Mix the medication in a Styrofoam cup. 4.Use a straw when giving the medication.

4 Tetracyclinbe is an antibiotic. Because tetracycline can cause staining of the teeth, straws should be used and the mouth rinsed after administration. The medication should be administered 1 hour before or 2 hours after the administration of milk, which would eliminate the options of giving the medication with milk or ice cream. Mixing the medication in a Styrofoam cup is unnecessary.

A client with acquired immunodeficiency syndrome has been started on therapy with zidovudine. The nurse assesses the complete blood cell (CBC) count, knowing that which is an adverse effect of this medication? 1.Polycythemia 2.Leukocytosis 3.Thrombocytosis 4.Agranulocytopenia

Acquired immunodeficiency syndrome is a viral disease caused by the human immunodeficiency virus (HIV), which destroys T cells, thereby increasing susceptibility to infection and malignancy. Zidovudine is a neucloside-nucleotide reverse transcriptase inhibitor used to the virus. An adverse effect of this medication is agranulocytopenia with anemia. The nurse carefully monitors CBC count results for changes that could indicate this occurrence. With early infection and in the client who is asymptomatic, the CBC count is monitored monthly for 3 months and then every 3 months thereafter. In clients with advanced disease, the CBC count is monitored every 2 weeks for the first 2 months and then once a month if the medication is tolerated well. The remaining options are not side or adverse effects of the medication.

The nurse is assigned to care for four clients. In planning client rounds, which client should the nurse assess first? 1.A postoperative client preparing for discharge with a new medication 2.A client requiring daily dressing changes of a recent surgical incision 3.A client scheduled for a chest x-ray after insertion of a nasogastric tube 4.A client with asthma who requested a breathing treatment during the previous shift

Airway is always the highest priority, and the nurse would attend to the client with asthma who requested a breathing treatment during the previous shift. This could indicate that the client was experiencing difficulty breathing. The clients described in options 1, 2, and 3 have needs that would be identified as intermediate priorities.

A client seen in the health care clinic is diagnosed with syphilis, and the health care provider prescribes an intramuscular injection of penicillin G benzathine. After administering the intramuscular injection of medication, the nurse should perform which action? 1.Monitor the client for 30 minutes. 2.Encourage the client to ambulate. 3.Administer subcutaneous epinephrine. 4.Apply a topical anesthetic spray to the injection site.

Penicillin G benzathine is an antibiotic. Anaphylactic shock is a possible reaction to penicillin therapy, and the onset of anaphylaxis nearly always occurs within 10 minutes. The client should be observed for 30 minutes after intramuscular injection so that if anaphylaxis develops, treatment is immediately available. Encouraging ambulation is unnecessary. The remaining 2 options are inaccurate interventions.

The nurse is preparing to administer phytonadione to the client. Which laboratory value should the nurse monitor in order to evaluate the effectiveness of the medication? 1.Prothrombin time 2.Blood ammonia level 3.Direct serum bilirubin 4.Serum potassium level

Phytonadione is needed for adequate blood clotting. Therefore, checking the prothrombin time is necessary 24 hours after injection of this medication. Blood ammonia levels are assessed to determine the conversion of ammonia to urea that normally occurs in the liver. Bilirubin is a measurement of the ability of the liver to conjugate and excrete bilirubin. Serum potassium is an electrolyte and is not affected by the injection of phytonadione.

A fluorescent antinuclear antibody titer (FANA) is performed in a client suspected of having rheumatoid arthritis (RA). Which laboratory value is most consistent with RA? 1.0:5 2.0:8 3.1:5 4.1:20

Rheumatoid arthritis is a chronic, progressive, systemic inflammatory autoimmune disease process that affects primarily the synovial joints. The antinuclear antibody test measures the titer of antibodies that destroy the nuclei cells and cause tissue death. When the fluorescent method is used, the test sometimes is referred to as FANA. If this test is positive, a value greater than 1:8 will be present. Therefore, the options below this value are incorrect.

A client is on nothing by mouth (NPO) status and has a nasogastric (NG) tube in place after suffering bilateral burns to the legs. The nurse determines that the client's gastrointestinal (GI) status is least satisfactory if which finding is noted on assessment? 1.Gastric pH of 3 2.Absence of abdominal discomfort 3.GI drainage that is guaiac negative 4.Presence of hypoactive bowel sounds

The gastric pH should be maintained at 7 or greater with the use of prescribed antacids and histamine 2 (H2) receptor-blocking agents. Lowered pH (to the acidic range) in the absence of food or tube feedings can lead to erosion of the gastric lining and ulcer development. Absence of discomfort and bleeding (guaiac-negative drainage) are normal findings. The client's bowel sounds may be expected to be hypoactive in the absence of oral or NG tube intake.

When creating an assignment for a team consisting of a registered nurse (RN), 1 licensed practical nurse (LPN), and 2 unlicensed assistive personnel (UAP), which is the best client for the LPN? 1.A client requiring frequent temperature checks 2.A client requiring assistance with ambulation every 4 hours 3.A client on a mechanical ventilator requiring frequent assessment and suctioning 4.A client with a spinal cord injury requiring urinary catheterization every 6 hours

When creating nursing assignments, the nurse needs to consider the skills and educational level of the nursing staff. Frequent temperature checks and ambulation can most appropriately be provided by the UAP, considering the clients identified in each option. The client on the mechanical ventilator requiring frequent assessment and suctioning should most appropriately be cared for by the RN. The LPN is skilled in urinary catheterization, so the client in option 4 would be assigned to this staff member.

Colchicine is prescribed for a client with a diagnosis of gout. The nurse reviews the client's record, knowing that this medication would be used with caution in which disorder? 1.Myxedema 2.Kidney disease 3.Hypothyroidism 4.Diabetes mellitus

2 Colchicine is used with caution in older clients, debilitated clients, and clients with cardiac, kidney, or gastrointestinal disease. The disorders in options 1, 3, and 4 are not concerns with administration of this medication.

An older client is diagnosed with osteoporosis. The nurse teaches the client about self-care measures, knowing that the client is most at risk for which problem as a result of this disorder of the bones? 1.Anemia 2.Fractures 3.Infection 4.Muscle sprains

2 The client is most at risk for fractures as a result of osteoporosis. Although other complications can occur, fracture is the greatest concern. Anemia and infection can occur with bone marrow disorders, and muscle sprains are unrelated to osteoporosis.

The nurse is caring for a client diagnosed with osteitis deformans (Paget's disease). Which does the nurse identify as the cause of the client's stooped posture and bowing of lower extremities? 1.Muscle metabolism and growth 2.Bone resorption and regeneration 3.Nervous system impulse transmission 4.Joint integrity and synovial fluid production

2 Paget's disease is characterized by skeletal deformities resulting from abnormal bone resorption followed by abnormal regeneration. It is not caused by problems with muscle, nervous system, or joint functioning.

The nurse teaches a client about the effects of diphenhydramine, which has been prescribed as a cough suppressant. The nurse determines that the client needs further instruction if the client makes which statement? 1."I will take the medication on an empty stomach." 2."I won't drink alcohol while taking this medication." 3."I won't do activities that require mental alertness while taking this medication." 4."I will use sugarless gum, candy, or oral rinses to decrease dryness in my mouth."

1 Diphenhydramine has several uses, including as an antihistamine, antitussive, antidyskinetic, and sedative-hypnotic. Instructions for use include taking with food or milk to decrease gastrointestinal upset and using oral rinses, sugarless gum, or hard candy to minimize dry mouth. Because the medication causes drowsiness, the client should avoid use of alcohol or central nervous system depressants, operating a car, or engaging in other activities requiring mental awareness during use.

A client's serum blood glucose level is 48 mg/dL (2.74 mmol/L). The nurse would expect to note which as an additional finding when assessing this client? 1.Slurred speech 2.Increased thirst 3.Increased appetite 4.Increased urination

1 A client who has a blood glucose level of less than 70 mg/dL (4 mmol/L) is considered to be hypoglycemic. A clinical manifestation of hypoglycemia is slurred speech.

The health care provider has prescribed a lidocaine 5% patch for a client with a diagnosis of neck pain due to osteoarthritis. Which should the nurse tell the client regarding this medication? 1.The medication patch will act as a local anesthetic. 2.The medication patch acts by decreasing muscle spasms. 3.The medication is prescribed to cause the skin to peel below the patch. 4.Apply a heating pad to the area after applying the medication patch to increase the effectiveness.

1 A lidocaine patch provides a local anesthetic effect to the site of application. The medication does not act in a systemic manner. It is not prescribed to cause the skin to peel, so if this reaction occurs, the health care provider should be notified. A heating pad should not be applied because irritation or burning of the skin may occur.

A client with a new medication prescription for allopurinol asks the nurse, "I know this is for gout, but how does it work?" The nurse plans to reply based on which medication action? 1.Allopurinol decreases uric acid production. 2.Allopurinol reduces the production of fibrinogen. 3.Allopurinol decreases the risk of sulfa crystal formation in the urine. 4.Allopurinol prevents influx of calcium ions during cell depolarization.

1 Allopurinol is classified as an antigout medication. It decreases uric acid production by inhibiting the xanthine oxidase enzyme, and it reduces uric acid concentrations in both serum and urine. The other options are incorrect.

A client has begun using a methylxanthine bronchodilator. What beverage should the nurse plan to teach the client to avoid while taking this medication? 1.Coffee 2.Orange juice 3.Mineral water 4.Cranberry juice

1 Cola, coffee, and chocolate contain methylxanthine and should be avoided by the client taking a methylxanthine bronchodilator. The additional methylxanthine could lead to increased incidence of cardiovascular and central nervous system side effects. Orange juice, mineral water, and cranberry juice are fluids that are allowed.

A client with diabetes mellitus is being discharged following treatment for hyperosmolar hyperglycemic syndrome (HHS) precipitated by acute illness. The client tells the nurse, "I will call the health care provider (HCP) the next time I can't eat for more than a day or so." Which statement reflects the most appropriate analysis of this client's level of knowledge? 1.The client needs immediate education before discharge. 2.The client requires follow-up teaching regarding the administration of oral antidiabetics. 3.The client's statement is inaccurate, and he or she should be scheduled for outpatient diabetic counseling. 4.The client's statement is inaccurate, and he or she should be scheduled for educational home health visits.

1 If the client becomes ill and cannot retain fluids or food for a period of 4 hours, the HCP should be notified. The client's statement indicates a need for immediate education to prevent hyperosmolar hyperglycemic syndrome (HHS), a life-threatening emergency. Although all of the other options may be true, the most appropriate analysis is that the client requires immediate education.

A client is admitted with a serum glucose level of 650 mg/dL (37.14 mmol/L) and diabetic ketoacidosis (DKA) is suspected. Which additional laboratory result does the nurse identify as being supportive of DKA? 1.Ketones in urine 2.Lactic dehydrogenase (LDH) of 200 U/L 3.pH of 7.52 on arterial blood gas (ABG) analysis 4.Blood urea nitrogen (BUN) of 10 mg/dL (3.6 mmol/L)

1 Ketones are a byproduct of fat metabolism. When this process occurs to the extreme, it is termed ketoacidosis. Ketone bodies are a product of fat metabolism, and the presence of moderate to high urine ketones (hyperketonuria) indicates a severe lack of insulin, such as in DKA. Options 2, 3, and 4 are incorrect. BUN of 10 mg/dL (3.6 mmol/L) is a normal value, as normal is 10 to 20 mg/dL (3.6 to 7.1 mmol/L). Elevations in LDH (normal is 100 to 190 U/L) can be indicative of heart failure, hemolytic disorders, hepatitis dysfunction, myocardial infarction, pulmonary embolus, or skeletal muscle damage. In acidotic conditions the pH will be decreased (normal is 7.35 to 7.45).

The nurse is performing an admission assessment on a client diagnosed with paronychia. The nurse should plan to assess which part of the integumentary system first? 1.Nails 2.Hair follicles 3.Pilosebaceous glands 4.Epithelial layer of skin

1 Paronychia is a fungal infection that most often is caused by Candida albicans. This results in inflammation of the nail fold, with separation of the fold from the nail plate. The affected area generally is tender to touch and has purulent drainage. Disorders of the hair follicles include folliculitis, furuncles, and carbuncles. Disorders of the pilosebaceous glands include acne vulgaris and seborrheic dermatitis. A variety of disorders may involve the epithelial skin layer.

The nurse is monitoring a client with Graves' disease for signs of thyrotoxic crisis (thyroid storm). Which signs or symptoms, if noted in the client, will alert the nurse to the presence of this crisis? 1.Fever and tachycardia 2.Pallor and tachycardia 3.Agitation and bradycardia 4.Restlessness and bradycardia

1 Thyrotoxic crisis (thyroid storm) is an acute, potentially life-threatening state of extreme thyroid activity that represents a breakdown in the body's tolerance to a chronic excess of thyroid hormones. The clinical manifestations include fever with temperatures greater than 100°F, severe tachycardia, flushing and sweating, and marked agitation and restlessness. Delirium and coma can occur.

The home health nurse visits a client with a diagnosis of type 1 diabetes mellitus. The client relates a history of vomiting and diarrhea and tells the nurse that no food has been consumed for the last 24 hours. Which additional statement by the client indicates a need for further teaching? 1."I need to stop my insulin." 2."I need to increase my fluid intake." 3."I need to monitor my blood glucose every 3 to 4 hours." 4."I need to call the health care provider (HCP) because of these symptoms."

1 When a client with diabetes mellitus is unable to eat normally because of illness, the client still should take the prescribed insulin or oral medication. The client should consume additional fluids and should notify the HCP. The client should monitor the blood glucose level every 3 to 4 hours. The client should also monitor the urine for ketones during illness.

The nurse is preparing a client for punch biopsy. What should the nurse do to prepare for this procedure? 1.Ensure that the consent form has been signed. 2.Ensure that a Foley catheter has been inserted. 3.Provide chlorhexidine wipes to be used before the procedure. 4.Verify the blood bank has 1 unit of packed red blood cells available if needed.

1 A punch biopsy involves use of a punch instrument that punctures the skin and is rotated to obtain some of the dermis and fat. It is used for diagnostic purposes. A signed consent form is required for this procedure. A Foley catheter is not indicated and should be avoided if possible for any condition or procedure due to the risk for catheter-associated urinary tract infection. Chlorhexidine wipes are not specifically indicated for this procedure; usually an antibacterial such as povidone-iodine is used. There is not typically a lot of bleeding with this procedure; therefore, units of blood are not typically made available for the client undergoing punch biopsy.

The nurse is caring for a client who has vesicles filled with purulent fluid on the face and upper extremities. On the basis of these findings, the nurse should tell the client that the vesicles are consistent with which condition? 1.Acne 2.Freckles 3.Psoriasis 4.Sebaceous cysts

1 Acne is characterized by vesicles filled with cloudy or purulent fluid. Freckles are flat lesions less than 1 centimeter. Psoriasis is presented by elevated, plateaulike patches more than 1 centimeter. Sebaceous cysts are nodules filled with either liquid or semisolid material that can be expressed.

The nurse has provided home care instructions to a client after blepharoplasty. Which statement by the client indicates a need for further instruction? 1."I need to keep ice on my eyes for at least 3 days." 2."I need to avoid vigorous activities for about 1 month." 3."I need to sleep on my back with at least 2 pillows under my head." 4."I need to avoid activities requiring bending over at the waist for at least 48 hours."

1 Blepharoplasty is the use of plastic surgery to restore or repair the eyelid or eyebrow (brow lift). Home care instructions after blepharoplasty include the administration of cool compresses for 24 (not 72) hours. Vigorous activities, such as sports, need to be avoided for 1 month. Because lying on the side increases the possibility of swelling in the dependent eye area, the client should sleep supine with at least 2 pillows to elevate the head. The client should understand the importance of not bending over at the waist for the first 48 hours after the procedure. Bending would increase pressure to the operative area.

The nurse is caring for a client who had surgery to repair a fractured left-sided hip using a posterior approach. In implementing hip precautions, which action should the nurse teach the client to avoid? 1.Crossing legs at the ankle 2.Using an elevated toilet seat 3.Placing a pillow between the legs 4.Keeping the legs abducted from the midline

1 Following surgery to repair a fractured hip using a posterior approach, client education should include the following: avoiding crossing the legs at the ankle or the knee, using an elevated toilet seat, placing a pillow between the legs while lying down for the first 6 weeks, keeping the legs abducted from the midline, and keeping the hip in a neutral position at all times.

The emergency department nurse is caring for a client who has sustained chemical burns to the esophagus after ingestion of lye. The nurse reviews the health care provider's prescriptions and should plan to question which prescription? 1.Gastric lavage 2.Intravenous (IV) fluid therapy 3.Nothing by mouth (NPO) status 4.Preparation for laboratory studies

1 The client who has sustained chemical burns to the esophagus is placed on NPO status, is given IV fluids for replacement and treatment of possible shock, and is prepared for esophagoscopy and barium swallow to determine the extent of damage. Laboratory studies also may be prescribed. A nasogastric tube may be inserted, but gastric lavage and emesis are avoided to prevent further erosion of the mucosa by the irritating substances that these treatments involve.

A client receiving oral theophylline is due to have a theophylline level drawn. The nurse should question the client to ensure that the client has not ingested which substance before the blood sample is drawn? 1.Coffee 2.Oatmeal 3.Ginger ale 4.Bagel with cream cheese

1 Theophylline is a xanthine bronchodilator. Before a serum level of the medication is drawn, the client should avoid taking foods or beverages that contain xanthine, such as colas, coffee, or chocolate; therefore, the client is told to avoid coffee before the test. The items in the other options do not need to be avoided before this test.

A client is hospitalized with a diagnosis of adrenal insufficiency. Which findings does the nurse identify as supportive of this diagnosis? Select all that apply. 1.Irritability 2.Complaints of nausea 3.Sodium level of 128 mEq/L (128 mmol/L) 4.Potassium level of 3.2 mEq/L (3.2 mmol/L) 5.Blood pressure lying 138/70 mm Hg and standing 110/58 mm Hg

1, 2, 3, 5 Findings consistent with a diagnosis of adrenal insufficiency include nausea, vomiting, and diarrhea; hyponatremia; salt craving; hyperkalemia; and orthostatic hypotension. Irritability and depression may also occur in primary adrenal hypofunction.

Rifabutin is prescribed for a client with active Mycobacterium avium complex (MAC) disease and tuberculosis. For which side and adverse effects of the medication should the nurse monitor? Select all that apply. 1.Signs of hepatitis 2.Flulike syndrome 3.Low neutrophil count 4.Vitamin B6 deficiency 5.Ocular pain or blurred vision 6.Tingling and numbness of the fingers

1, 2, 3, 5 Rifabutin may be prescribed for a client with active MAC disease and tuberculosis. It inhibits mycobacterial DNA-dependent RNA polymerase and suppresses protein synthesis. Side and adverse effects include rash, gastrointestinal disturbances, neutropenia (low neutrophil count), red-orange-colored body secretions, uveitis (blurred vision and eye pain), myositis, arthralgia, hepatitis, chest pain with dyspnea, and flulike syndrome. Vitamin B6 deficiency and numbness and tingling in the extremities are associated with the use of isoniazid.

A client with chronic obstructive pulmonary disease (COPD) is being evaluated for lung transplantation. The nurse performs the initial physical assessment. Which findings should the nurse anticipate in this client? Select all that apply. 1.Dyspnea at rest 2.Clubbed fingers 3.Muscle retractions 4.Decreased respiratory rate 5.Increased body temperature 6.Prolonged expiratory breathing phase

1, 2, 3, 6 The client with COPD who is eligible for a lung transplantation has end-stage COPD and will have clinical manifestations of hypoxemia, dyspnea at rest, use of accessory muscle with retractions, clubbing, and prolonged expiratory breathing phase caused by retention of carbon dioxide. Option 4 is not correct because the client with COPD has an increased respiratory rate, not a decreased one. Option 5 is not correct because an elevated temperature would not be present unless the client has an infection.

A client has been diagnosed with hyperthyroidism. The nurse monitors for which signs and symptoms indicating a complication of this disorder? Select all that apply. 1.Fever 2.Nausea 3.Lethargy 4.Tremors 5.Confusion 6.Bradycardia

1, 2, 4, 5 Thyroid storm is an acute and life-threatening complication that occurs in a client with uncontrollable hyperthyroidism. Signs and symptoms of thyroid storm include elevated temperature (fever), nausea, and tremors. In addition, as the condition progresses, the client becomes confused. The client is restless and anxious and experiences tachycardia.

The nurse in the ambulatory care clinic is reviewing a plan of care for a client who will be returning from the postanesthesia care unit after a blepharoplasty. Which nursing interventions should be a component of the postoperative care plan for this client? Select all that apply. 1.Monitoring for swelling 2.Elevating the head of the bed 3.Applying warm gauze pads to the eyes 4.Instructing the client to avoid Valsalva maneuvers 5.Assessing the function of the extraocular eye muscles

1, 2, 4, 5 Blepharoplasty is the use of plastic surgery to restore or repair the eyelid or eyebrow (brow lift). Postoperatively, the client is assessed for swelling, bruising, bleeding, and eye pain. The head of the bed should be elevated, and cool eye compresses are applied to the area to reduce swelling. The client is instructed to avoid the Valsalva maneuver, which increases intracranial pressure and also pressure in the head and eye, thereby increasing the risk of hemorrhage. The function of extraocular eye muscles also is assessed. Gauze pads are not used because cotton is thick and pulls the skin when it is removed; in addition, warm compresses will increase the swelling.

The nurse should provide which home care instructions to a client who had a laryngectomy and has a stoma? Select all that apply. 1.Increase the humidity in the home. 2.Obtain and wear a MedicAlert bracelet. 3.Wear clothing that does not cover the stoma. 4.Stay away from people who have a respiratory infection. 5.Be careful with showering to avoid water entering the stoma. 6.Decrease fluid intake to prevent excessive secretions from the stoma.

1, 2, 4, 5 The nurse should teach the client how to care for the stoma, depending on the type of laryngectomy performed. Most interventions focus on protection of the stoma and the prevention of infection. Interventions include obtaining a MedicAlert bracelet, avoiding exposure to people with infections, avoiding swimming, using care when showering, and preventing debris from entering the stoma. Additional interventions include wearing a stoma guard or high-collared clothing to cover the stoma, increasing the humidity in the home, and increasing fluid intake to 3000 mL/day to keep the secretions thin.

The nurse is teaching the client about risk factors for skin cancer. Which statements by the client indicate that teaching was successful? Select all that apply. 1."I have to avoid excessive exposure to sunlight." 2."My dark skin color predisposes me to skin cancer." 3."I am at higher risk for skin cancer because my mother had one." 4."I am at higher risk for skin cancer because I am 20 years old." 5."I am immune to skin cancer because I work as a pest control exterminator."

1, 3 Options 1 and 3 describe risk factors for skin cancer. Additional risk factors for skin cancer include age greater than 60 years, light-colored skin, and occupation exposure to arsenic, which is commonly used in pest control.

An older client has been lying in a supine position for the past 3 hours. The nurse who is repositioning this client would be most concerned with examining which bony prominences of the client? Select all that apply. 1.Heels 2.Ankles 3.Elbows 4.Sacrum 5.Back of the head 6.Greater trochanter

1, 3, 4, 5 When the client is lying supine, the heels, sacrum, and back of the head are all at risk, as are the elbows and scapulae. The greater trochanter and ankles are at greater risk of skin breakdown from excessive pressure when the client is in the side-lying position.

The nurse is admitting a client who is diagnosed with syndrome of inappropriate antidiuretic hormone secretion (SIADH) and has serum sodium of 118 mEq/L (118 mmol/L). Which health care provider prescriptions should the nurse anticipate receiving? Select all that apply. 1.Initiate an infusion of 3% NaCl. 2.Administer intravenous furosemide. 3.Restrict fluids to 800 mL over 24 hours. 4.Elevate the head of the bed to high Fowler's. 5.Administer a vasopressin antagonist as prescribed.

1, 3, 5 Clients with SIADH experience excess secretion of antidiuretic hormone (ADH), which leads to excess intravascular volume, a declining serum osmolarity, and dilutional hyponatremia. Management is directed at correcting the hyponatremia and preventing cerebral edema. Hypertonic saline is prescribed when the hyponatremia is severe, less than 120 mEq/L (120 mmol/L). An intravenous (IV) infusion of 3% saline is hypertonic. Hypertonic saline must be infused slowly as prescribed and an infusion pump must be used. Fluid restriction is a useful strategy aimed at correcting dilutional hyponatremia. Vasopressin is an ADH; vasopressin antagonists are used to treat SIADH. Furosemide may be used to treat extravascular volume and dilutional hyponatremia in SIADH, but it is only safe to use if the serum sodium is at least 125 mEq/L (125 mmol/L). When furosemide is used, potassium supplementation should also occur and serum potassium levels should be monitored. To promote venous return, the head of the bed should not be raised more than 10 degrees for the client with SIADH. Maximizing venous return helps to avoid stimulating stretch receptors in the heart that signal to the pituitary that more ADH is needed.

The nurse is caring for a client with acute back pain. Which are the most likely causes of this problem? Select all that apply. 1.Twisting of the spine 2.Curvature of the spine 3.Hyperflexion of the spine 4.Sciatic nerve inflammation 5.Degeneration of the facet joints 6.Herniation of an intervertebral disk

1, 3, 6 Acute back pain is sudden in onset and is usually precipitated by injury to the lower back, such as with hyperflexion, twisting, or disk herniation. Scoliosis (curvature), sciatica, and degenerative vertebral changes are more likely to cause chronic back pain, which can be more insidious in onset and may also be accompanied by degeneration of the intervertebral disk.

The nurse teaches a class on foot care for clients diagnosed with diabetes mellitus. Which instructions should the nurse include in the class? Select all that apply. 1.Wear closed-toe shoes. 2.Soak feet in hot water twice a day. 3.Massage lanolin lotion between the toes. 4.Cut toenails straight across and file the edges. 5.Pat feet dry gently, especially between the toes.

1, 4, 5 People with diabetes mellitus are at high risk for foot ulcerations and resultant lower extremity amputations. The development of diabetic foot complications can be the result of a combination of microvascular and macrovascular diseases that place the client at risk for injury and serious infection. Options 1, 4, and 5 are correct, as measures should be taken to teach clients how to prevent foot ulcers and injury. These measures include wearing closed-toe shoes to protect the feet and toes (especially for those with peripheral neuropathy), cutting toenails straight across and filing the edges to avoid sharp toenail edges and cutting the skin of the toe, and drying the feet gently and thoroughly (including between the toes) to prevent maceration of the skin and infections. Option 2 is incorrect, as clients with diabetes should avoid hot water due to neuropathy and possible burns. Option 3 is incorrect, as lotion between the toes is not advised; it is necessary to keep the area between the toes dry to avoid maceration and infections.

A client is returning from surgery after a pulmonary lobectomy. Which pieces of equipment should the nurse have at the bedside? Select all that apply. 1.Clamp 2.Code cart 3.Central line kit 4.Vaseline gauze 5.Tracheotomy set 6.Suction equipment

1, 4, 6 The nurse should anticipate that a client with a lobectomy will have a chest tube and will need suction, Vaseline gauze, and a clamp at the bedside for emergency use. The nurse would not need a code cart at the bedside unless the client was in cardiac arrest. A central line kit at the bedside does not apply to the situation. A tracheotomy set is not necessary.

A client with diabetes mellitus demonstrates acute anxiety when admitted to the hospital for the treatment of hyperglycemia. What is the appropriate intervention to decrease the client's anxiety? 1.Administer a sedative. 2.Convey empathy, trust, and respect toward the client. 3.Ignore the signs and symptoms of anxiety, anticipating that they will soon disappear. 4.Make sure that the client is familiar with the correct medical terms to promote understanding of what is happening.

2 Anxiety is a subjective feeling of apprehension, uneasiness, or dread. The appropriate intervention is to address the client's feelings related to the anxiety. Administering a sedative is not the most appropriate intervention and does not address the source of the client's anxiety. The nurse should not ignore the client's anxious feelings. Anxiety needs to be managed bef

A client taking rifampin reports, "My urine has blood in it." When the nurse assesses the urine, it is brown. Which is the nurse's best action? 1.Notify the health care provider (HCP). 2.Chart the finding as a normal response to the rifampin. 3.Immediately start prescribed intravenous (IV) fluids to prevent shock. 4.Get the client into bed, and put the bed in modified Trendelenburg's position.

2 Brown-tinged urine is a normal finding associated with rifampin; thus, there is no need to notify the HCP. There is no indication that the client is in shock, so eliminate the options that indicate to start prescribed IV fluids and to place the client in modified Trendelenburg's position. The nurse should also inform the client that his is a harmless side effect.

A client with type 1 diabetes mellitus calls the nurse to report recurrent episodes of hypoglycemia with exercising. Which statement by the client indicates an adequate understanding of the peak action of NPH insulin and exercise? 1."I should not exercise since I am taking insulin." 2."The best time for me to exercise is after breakfast." 3."The best time for me to exercise is mid- to late afternoon." 4."NPH is a basal insulin, so I should exercise in the evening."

2 Exercise is an important part of diabetes management. It promotes weight loss, decreases insulin resistance, and helps to control blood glucose levels. A hypoglycemic reaction may occur in response to increased exercise, so clients should exercise either an hour after mealtime or after consuming a 10- to 15-gram carbohydrate snack, and they should check their blood glucose level before exercising. Option 1 is incorrect because clients with diabetes should exercise, though they should check with their health care provider before starting a new exercise program. Option 3 in incorrect; clients should avoid exercise during the peak time of insulin. NPH insulin peaks at 4 to 12 hours; therefore, afternoon exercise takes place during the peak of the medication. Options 4 is incorrect; NPH insulin in an intermediate-acting insulin, not a basal insulin.

The nurse is performing an assessment on a client with a diagnosis of hyperthyroidism. Which assessment finding should the nurse expect to note in this client? 1.Dry skin 2.Bulging eyeballs 3.Periorbital edema 4.Coarse facial features

2 Hyperthyroidism is clinically manifested by goiter (increase in the size of the thyroid gland) and exophthalmos (bulging eyeballs). Other clinical manifestations include nervousness, fatigue, weight loss, muscle cramps, and heat intolerance. Additional signs found in this disorder include tachycardia; shortness of breath; excessive sweating; fine muscle tremors; thin, silky hair and thin skin; infrequent blinking; and a staring appearance.

The nurse caring for a client with a diagnosis of hypoparathyroidism reviews the laboratory results of blood tests for this client and notes that the calcium level is extremely low. The nurse should expect to note which finding on assessment of the client? 1.Unresponsive pupils 2.Positive Trousseau's sign 3.Negative Chvostek's sign 4.Hypoactive bowel sounds

2 Hypoparathyroidism is related to a lack of parathyroid hormone secretion or a decreased effectiveness of parathyroid hormone on target tissues. The end result of this disorder is hypocalcemia. When serum calcium levels are critically low, the client may exhibit Chvostek's and Trousseau's signs, which indicate potential tetany. The remaining options are not related to the presence of hypocalcemia.

The nurse is planning care for a client who suffered a burn injury and has a negative self-image related to keloid formation at the burn site. The keloid formation is indicative of which condition? 1.Nerve damage 2.Hypertrophy of collagen fibers 3.Compromised circulation at the burn site 4.Increase in subcutaneous tissue at the burn site

2 Keloids are visible as excessive scar formation and result from hypertrophy of collagen fibers. Nerves conduct sensory and motor impulses from the skin. The vasculature provides blood vessels with nourishment and assists in thermoregulation. Subcutaneous tissue provides for heat insulation, mechanical shock absorption, and caloric reserve.

The nurse is caring for a client who sustained an open fracture and is diagnosed with acute osteomyelitis of the right lower extremity. Which intervention should the nurse plan to perform? 1.Apply ice to the affected area. 2.Perform sterile dressing changes. 3.Instruct the client on leg exercises. 4.Measure the leg circumference daily.

2 Osteomyelitis is a severe infection of the bone, bone marrow, and surrounding soft tissue. Clinical manifestations include constant bone pain unrelieved by rest that worsens with activity; swelling, tenderness, and warmth at the infection site; restricted movement of the affected part; fever, night sweats, chills, restlessness, nausea, and malaise. Option 2 is the correct option, as treatment of osteomyelitis often includes surgical debridement and requires sterile dressing changes. Option 1 is incorrect, as osteomyelitis is an infection and applying ice to the area will not help any swelling and may cause vasoconstriction. Option 3 is incorrect, as movement worsens the pain and some immobilization of the affected limb (e.g., splint, traction) is usually indicated. Option 4, measuring leg circumference daily, is not necessary.

Diagnostic studies are prescribed for a client with suspected Paget's disease. In reviewing the client's record, the nurse would expect to note that the health care provider has prescribed which laboratory study? 1.Platelet count 2.Alkaline phosphatase 3.White blood cell count 4.Complete blood cell count

2 Paget's disease is a chronic metabolic disorder in which bone is excessively broken down and reformed. The result is bone that is structurally disorganized, causing bone to be weak with increased risk for bowing of long bones and fractures. Diagnostic laboratory findings for Paget's disease include an elevated serum alkaline phosphatase level and elevated urinary hydroxyproline excretion. The remaining options are unrelated to diagnostic evaluation of this disease.

The nurse has provided discharge instructions to the client who has had a pneumonectomy. Which statement, if made by the client, indicates an understanding of appropriate home care measures? 1."I should restrict my fluid intake for 2 weeks." 2."I should perform arm exercises 2 or 3 times a day." 3."If I experience any soreness in my chest or shoulder, I should notify the health care provider." 4."If I experience any numbness or altered sensation around the incision, I should contact the health care provider."

2 The client should be instructed to perform arm and shoulder exercises 2 or 3 times a day to prevent frozen shoulder. The client is encouraged to drink liquids to liquefy secretions, making them easier to expectorate. The client is told to expect soreness in the chest and shoulder and an altered feeling of sensation around the incision site for several weeks. It is not necessary to contact the health care provider if these symptoms occur.

A client who experiences frequent upper respiratory infections (URIs) asks the nurse why food does not seem to have any taste during illness. Which response by the nurse is most appropriate? 1."You lack the energy to cook wholesome meals." 2."Blocked nasal passages impair the sense of smell." 3."Loss of appetite is triggered by the infectious organism." 4."Infection blocks sensation in the taste buds of the tongue."

2 When nasal passages become blocked as a result of a URI, the client has an impaired sense of taste and smell. This occurs because one of the normal functions of the nose is to stimulate appetite through the sense of smell. The other options are incorrect and unrelated to this symptom.

The nurse is repositioning a client who has been returned to the nursing unit after internal fixation of a fractured right hip with a femoral head replacement. The nurse should use which method to reposition the client? 1.A trochanter roll to prevent abduction during turning 2.A pillow to keep the right leg abducted during turning 3.A pillow to keep the right leg adducted during turning 4.A trochanter roll to prevent external rotation during turning

2 After femoral head replacement for a fractured hip with an intracapsular fracture, the client is turned to the affected side or the unaffected side as prescribed by the surgeon. Before moving the client, the nurse places a pillow between the client's legs to keep the affected leg in abduction. The nurse then repositions the client while maintaining proper alignment and abduction. A trochanter roll is useful in preventing external rotation, but it is used after the client has been repositioned. A trochanter roll is not used while the client is being turned.

The nurse has been working with the client diagnosed with candidiasis (thrush). What should the nurse assess for in this client? 1.The presence of blisters 2.The presence of white patches 3.The presence of purple patches 4.The presence of numerous small, red, pinpoint lesions

2 Assessment of the client with candidiasis (thrush) will reveal white patches on the tongue, palate, and buccal mucosa. The lesions adhere firmly to the tissues and are difficult to remove. The lesions often are referred to as "milk curds" because of their appearance. Clients often describe the lesions as dry and hot. Options 1, 3, and 4 are not characteristics of thrush.

A client scheduled for a skin biopsy is concerned and asks the nurse how painful the procedure is. Which statement is the appropriate response by the nurse? 1."There is no pain associated with this procedure." 2."The local anesthetic may cause a burning or stinging sensation." 3."A preoperative medication will be given so you will be sleeping and will not feel any pain." 4."There is some pain, but the health care provider will prescribe an opioid analgesic after the procedure."

2 Depending on the size and location of the lesion, a biopsy is usually a quick and almost painless procedure. The most common source of pain is the administration of the initial local anesthetic, which can produce a burning or stinging sensation. Preoperative medication is not necessary with this procedure. Opioid analgesics are not necessary following the procedure.

A client has been on treatment for rheumatoid arthritis for 3 weeks. During the administration of etanercept, which is most important for the nurse to assess? 1.The injection site for itching and edema 2.The white blood cell counts and platelet counts 3.Whether the client is experiencing fatigue and joint pain 4.Whether the client is experiencing a metallic taste in the mouth, and a loss of appetite

2 Infection and pancytopenia are adverse effects of etanercept. Laboratory studies are performed prior to and during medication treatment. The appearance of abnormal white blood cell counts and abnormal platelet counts can alert the nurse to a potentially life-threatening infection. Injection site itching is a common occurrence following administration. A metallic taste and loss of appetite are not common signs of adverse effects of this medication.

A client with a 4-day-old lumbar vertebral fracture is experiencing muscle spasms. Which are interventions to aid the client in relieving the spasm? Select all that apply. 1.Ice 2.Heat 3.Analgesics 4.Muscle relaxers 5.Intermittent traction

2, 3, 4, 5 Heat, analgesics, muscle relaxers, and traction all may be used to relieve the pain of muscle spasm in the client with a vertebral fracture. Ice is applied to a painful site only for the first 48 to 72 hours (depending on the health care provider's preference) after an injury. Application of ice to the spine of a client could be uncomfortable and could result in feelings of being chilled.

The client questions the nurse as to why the health care provider switched the usual prescription from a metered-dose inhaler (MDI) to a dry powder inhaler (DPI). The nurse should respond correctly by providing which facts? Select all that apply. 1.Dry powder inhalers have fewer side effects. 2.Dry powder inhalers pose no environmental risks. 3.Dry powder inhalers can be administered more frequently. 4.Dry powder inhalers deliver more medication to the lungs. 5.Dry powder inhalers require less hand-to-lung coordination.

2, 4, 5 DPIs are used to deliver medications in the form of a dry, micronized powder directly to the lungs. DPIs do not require the hand-to-lung coordination needed with MDIs; thus, DPIs are much easier to use. Compared with MDIs, DPIs deliver more medication to the lungs (20% of the total released versus 10%) and less to the oropharynx. Because DPIs do not require propellant, they are not a risk to the environment. Both types of inhalers have side effects. Frequency of use is prescribed by the health care provider.

A client taking albuterol by inhalation cannot cough up secretions. What should the nurse suggest that the client do to assist in expectoration of secretions? 1.Get more exercise each day. 2.Use a dehumidifier in the home. 3.Drink increased amounts of fluids every day. 4.Take an extra dose of albuterol before bedtime.

3 A client should drink increased fluids (2000 to 3000 mL/day) to decrease viscosity and increase expectoration of secretions. This is standard advice for clients receiving any of the adrenergic bronchodilators, unless the client has another health problem that contraindicates an increased fluid intake. Additional exercise will not effectively clear bronchial secretions. A dehumidifier will dry secretions, making the situation worse. The client should not take additional medication.

The nurse is monitoring a client for signs of hypocalcemia after thyroidectomy. Which sign or symptom, if noted in the client, would most likely indicate the presence of hypocalcemia? 1.Bradycardia 2.Flaccid paralysis 3.Tingling around the mouth 4.Absence of Chvostek's sign

3 After thyroidectomy the nurse assesses the client for signs of hypocalcemia and tetany. Early signs include tingling around the mouth and in the fingertips, muscle twitching or spasms, palpitations or arrhythmias, and Chvostek's and Trousseau's signs. Bradycardia, flaccid paralysis, and absence of Chvostek's sign are not signs of hypocalcemia.

The nurse is instructing a client in diaphragmatic breathing. To reinforce the need for this technique, the nurse teaches the client that in normal respiration, which is an action of the diaphragm? 1.Aids in exhalation as it contracts 2.Moves up and inward as it contracts 3.Moves downward and out as it contracts 4.Makes the thoracic cage smaller as it contracts

3 As the diaphragm contracts it moves downward and out, becoming flatter and expanding the thoracic cage. This process occurs during the inspiratory phase of the respiratory cycle. Therefore, the remaining options are incorrect.

he nurse is performing assessment of the client who is admitted with left leg cellulitis. What does the nurse anticipate finding on the assessment of the left lower extremity? 1.Pallor 2.Cyanosis 3.Erythema 4.Jaundice

3 Cellulitis presents with erythema (redness), which is localized inflammation. Options 1, 2, and 4 are not signs or symptoms of cellulitis.

The nurse is caring for a client who is on strict bed rest and creates a plan of care with goals related to the prevention of deep vein thrombosis and pulmonary emboli. Which nursing action is most helpful in preventing these disorders from developing? 1.Restricting fluids 2.Placing a pillow under the knees 3.Encouraging active range-of-motion exercises 4.Applying a heating pad to the lower extremities

3 Clients at greatest risk for deep vein thrombosis and pulmonary emboli are immobilized clients. Basic preventive measures include early ambulation, leg elevation, active leg exercises, elastic stockings, and intermittent pneumatic calf compression. Keeping the client well hydrated is essential because dehydration predisposes to clotting. A pillow under the knees may cause venous stasis. Heat should not be applied without a health care provider's prescription.

The nurse participating in a client care conference with other health team members is discussing the condition of a client with adult respiratory distress syndrome (ARDS). The health care provider states that because of fluid in the alveoli, surfactant production is falling. The nurse anticipates that insufficient surfactant will cause which effect? 1.Atelectasis and viral infection 2.Bronchoconstriction and stridor 3.Collapse of alveoli and decreased compliance 4.Decreased ciliary action and retained secretions

3 Surfactant is a phospholipid produced in the lungs that decreases surface tension in the lungs. This prevents the alveoli from sticking together and collapsing at the end of exhalation. When alveoli collapse, the lungs become "stiff" because of decreased compliance. Common causes of decreased surfactant production are ARDS and atelectasis. Viral infection may be one reason a client develops atelectasis. The remaining options are incorrect.

The nurse is performing nasotracheal suctioning of a client. The nurse determines that the client is tolerating the procedure if which observation is made? 1.The skin color becomes cyanotic. 2.Secretions are becoming bloody. 3.Coughing occurs with suctioning. 4.Heart rate decreases from 78 to 54 beats/minute.

3 The nurse monitors for adverse effects of suctioning, which include cyanosis, excessively rapid or slow heart rate, and sudden development of bloody secretions. If any of these signs is observed, the nurse immediately stops suctioning and reports the adverse effect to the health care provider. Coughing is a normal response to suctioning for the client with an intact cough reflex and does not indicate that he or she cannot tolerate the procedure.

A client who has been receiving theophylline by the intravenous (IV) route has the medication prescription changed to an immediate-release oral form of the medication. After discontinuing the IV medication, when should the nurse schedule the first dose of the oral medication? 1.Just after the next meal 2.Just before the next meal 3.4 hours after discontinuing the IV form 4.Immediately on discontinuing the IV form

3 With immediate-release preparations, oral theophylline should be administered 4 to 6 hours after discontinuing the IV form of the medication. If the sustained-release form is used, the first oral dose should be administered immediately on discontinuation of the IV infusion. Therefore, the remaining options are incorrect.

Zafirlukast is prescribed for a client with bronchial asthma. Which laboratory test does the nurse expect to be prescribed before the administration of this medication? 1.Platelet count 2.Neutrophil count 3.Liver function tests 4.Complete blood count

3 Zafirlukast is a leukotriene receptor antagonist used in the prophylaxis and long-term treatment of bronchial asthma. Zafirlukast is used with caution in clients with impaired hepatic function. Liver function laboratory tests should be performed to obtain a baseline, and the levels should be monitored during administration of the medication. It is not necessary to perform the other laboratory tests before administration of the medication.

The nurse is assessing the respiratory status of a client who has suffered a fractured rib. The nurse should expect to note which finding? 1.Slow, deep respirations 2.Rapid, deep respirations 3.Paradoxical respirations 4.Pain, especially with inspiration

4 Rib fractures result from a blunt injury or a fall. Typical signs and symptoms include pain and tenderness localized at the fracture site that is exacerbated by inspiration and palpation, shallow respirations, splinting or guarding the chest protectively to minimize chest movement, and possible bruising at the fracture site. Paradoxical respirations are seen with flail chest.

The nurse has completed giving discharge instructions to a client who has had a total joint replacement (TJR) of the knee with a metal prosthetic system. The nurse determines that the client understands the instructions if the client makes which statement? 1."Changes in the shape of the knee are expected." 2."Fever, redness, and increased pain are expected." 3."All caregivers should be told about the metal implant." 4."Bleeding gums or black stools may occur, but this is normal."

3 A TJR is also known as a total joint arthroplasty (TJA). The client must inform other caregivers of the presence of the metal implant because certain tests and procedures will need to be avoided. After total knee replacement, the client should report signs and symptoms of infection and any changes in the shape of the knee. These could indicate developing complications. With a metal implant, the client may be on anticoagulant therapy and should report adverse effects of this therapy, including bleeding from a variety of sources, and the client will need antibiotic prophylaxis for invasive procedures.

An adult client was burned in an explosion. The burn initially affected the client's entire face (anterior half of the head) and the upper half of the anterior torso, and there were circumferential burns to the lower half of both arms. The client's clothes caught on fire, and the client ran, causing subsequent burn injuries to the posterior surface of the head and the upper half of the posterior torso. Using the rule of nines, what would be the extent of the burn injury? 1.18% 2.24% 3.36% 4.48%

3 According to the rule of nines, with the initial burn, the anterior half of the head equals 4.5%, the upper half of the anterior torso equals 9%, and the lower half of both arms equals 9%. The subsequent burn included the posterior half of the head, equaling 4.5%, and the upper half of posterior torso, equaling 9%. This totals 36%.

A client sustained a burn from cutaneous exposure to lye. At the site of injury, copious irrigation to the site was performed for 1 hour. On admission to the hospital emergency department, the nurse assesses the burn site. Which findings would indicate that the chemical burn process is continuing? 1.Eschar 2.Intact blisters 3.Liquefaction necrosis 4.Cherry-red, firm tissue

3 Alkalis, such as lye, cause a liquefaction necrosis, and exposure to fat results in formation of a soapy coagulum. Thick, leathery eschar forms with exposure to acids or heat. Intact blisters indicate a partial-thickness thermal injury. Cherry-red, firm tissue can occur as a result of thermal injury.

The nurse is caring for a client following an autograft and grafting to a burn wound on the right knee. What would the nurse anticipate to be prescribed for the client? 1.Out-of-bed activities 2.Bathroom privileges 3.Immobilization of the affected leg 4.Placing the affected leg in a dependent position

3 Autografts placed over joints or on the lower extremities after surgery often are elevated and immobilized for 3 to 7 days. This period of immobilization allows the autograft time to adhere to the wound bed. Getting out of bed, going to the bathroom, and placing the grafted leg dependent would put stress on the grafted wound.

A client has just been admitted to the hospital with a fractured femur and pelvic fractures. The nurse should plan to carefully monitor the client for which signs/symptoms? 1.Fever and bradycardia 2.Fever and hypertension 3.Tachycardia and hypotension 4.Bradycardia and hypertension

3 Clients who experience fractures of the femur, pelvis, thorax, and spine are at risk for hypovolemic shock. Bone fragments can damage blood vessels, leading to hemorrhage into the abdominal cavity and the thigh. This can occur with closed fractures as well as open fractures. Signs of hypovolemic shock include tachycardia and hypotension.

The nurse is caring for a client admitted for a herniated intervertebral lumbar disk who is complaining about stabbing pain radiating to the lower back and the right buttock. The nurse determines that the client's signs/symptoms are most likely due to which condition? 1.Pressure on the spinal cord 2.Pressure on the spinal nerve root 3.Muscle spasm in the area of the herniated disk 4.Excess cerebrospinal fluid production in the area

3 Compression of a nerve results in inflammation, which then irritates adjacent muscles, putting them into spasm. The pain of muscle spasm is continuous, knife-like, and localized in the affected area. Pressure on the spinal cord itself could result in a variety of manifestations, depending on the area involved. Pressure on a spinal nerve root causes the symptoms of sciatica.

A client is admitted to the hospital with a diagnosis of pheochromocytoma. The nurse would check which item to detect the primary manifestation of this disorder? 1.Weight 2.Urine ketones 3.Blood pressure 4.Skin temperature

3 Hypertension is the major symptom associated with pheochromocytoma and is assessed by taking the client's blood pressure. Glycosuria, weight loss, and diaphoresis are other clinical manifestations of pheochromocytoma; however, hypertension is the major symptom.

The nurse performs a physical assessment on a client with type 2 diabetes mellitus. Findings include a fasting blood glucose level of 120 mg/dL (6.8 mmol/L), temperature of 101°F (38.3°C), pulse of 102 beats/minute, respirations of 22 breaths/minute, and blood pressure of 142/72 mm Hg. Which finding would be the priority concern to the nurse? 1.Pulse 2.Respiration 3.Temperature 4.Blood pressure

3 In the client with type 2 diabetes mellitus, an elevated temperature may indicate infection. Infection is a leading cause of hyperosmolar hyperglycemic syndrome in the client with type 2 diabetes mellitus. The other findings are within normal limits.

The nurse is administering an intravenous dose of methocarbamol to a client with multiple sclerosis. For which adverse effect should the nurse monitor? 1.Tachycardia 2.Rapid pulse 3.Bradycardia 4.Hypertension

3 Intravenous administration of methocarbamol can cause hypotension and bradycardia. The nurse needs to monitor for these adverse effects. Options 1, 2, and 4 are not effects with administration of this medication.

A nursing student is developing a plan of care for a client with a chest tube that is attached to a chest drainage system. Which intervention in the care plan indicates the need for further teaching for the student? 1.Position the client in semi Fowler's position. 2.Add water to the suction chamber as it evaporates. 3.Instruct the client to avoid coughing and deep breathing. 4.Tape the connection sites between the chest tube and the drainage system.

3 It is important to encourage the client to cough and deep breathe when a chest tube drainage system is in place. This will assist in facilitating appropriate lung reexpansion. The client is positioned in semi Fowler's position to facilitate ease in breathing. Water is added to the suction chamber as it evaporates to maintain the full suction level prescribed. Connections between the chest tube and the drainage system are taped to prevent accidental disconnection.

The nurse is reviewing the postprocedure plan of care formulated by a nursing student for a client scheduled for a bone biopsy. The nurse determines that the student needs additional information about postprocedure care if which inaccurate intervention is documented? 1.Elevating the limb 2.Monitoring vital signs every 4 hours 3.Administering opioid analgesics intramuscularly 4.Monitoring the biopsy site for swelling, bleeding, or hematoma

3 Nursing care after bone biopsy includes monitoring the site for swelling, bleeding, and hematoma formation. The biopsy site is elevated for 24 hours or as prescribed to reduce edema. The vital signs are monitored every 4 hours for 24 hours for signs of complications such as infection and bleeding. The client usually requires mild analgesics. More severe pain usually indicates that complications are arising.

The nurse is caring for the client who has skeletal traction applied to the left leg. The client complains of severe left leg pain. The nurse checks the client's alignment in bed and notes that proper alignment is maintained. Which is the priority nursing action? 1.Provide pin care. 2.Medicate the client. 3.Call the health care provider. 4.Remove 2 pounds (0.9 kg) of weight from the traction system.

3 Severe pain in a client in skeletal traction may indicate a need for realignment, or the traction weights applied to the limb may be too heavy. The nurse realigns the client. If this measure is ineffective, the nurse then calls the health care provider. Severe leg pain once traction has been established indicates a problem. Providing pin care is unrelated to the problem as described. Medicating the client should be done after trying to determine and treat the cause. The nurse should never remove the weights from the traction system without a specific prescription to do so.

The nurse is monitoring a client with diabetes mellitus for signs of hypoglycemia. Which manifestations are associated with this complication? 1.Slow pulse; lethargy; warm, dry skin 2.Elevated pulse; lethargy; warm, dry skin 3.Elevated pulse; shakiness; cool, clammy skin 4.Slow pulse, confusion, increased urine output

3 Signs and symptoms of mild hypoglycemia include tachycardia; shakiness; and cool, clammy skin. (+slurred speech). The remaining options do not specify the manifestations of hypoglycemia.

The nurse is assessing the casted extremity of a client. Which sign is indicative of infection? 1.Dependent edema 2.Diminished distal pulse 3.Presence of a "hot spot" on the cast 4.Coolness and pallor of the extremity

3 Signs of infection under a casted area include odor or purulent drainage from the cast or the presence of "hot spots," which are areas of the cast that are warmer than others. The health care provider should be notified if any of these occur. Signs of impaired circulation in the distal limb include coolness and pallor of the skin, diminished distal pulse, and edema.

The nurse is planning measures to increase bed mobility for a client in skeletal leg traction. Which item should the nurse consider to be most helpful for this client? 1.Television 2.Fracture bedpan 3.Overhead trapeze 4.Reading materials

3 The use of an overhead trapeze is extremely helpful for a client to move about in bed and to get on and off the bedpan. This device has the greatest value in increasing overall bed mobility. Television and reading materials, although helpful in reducing boredom and providing distraction, do not increase bed mobility. A fracture bedpan is useful in reducing discomfort with elimination.

A client calls the emergency department and tells the nurse that he came directly into contact with poison ivy shrubs. The client tells the nurse that he cannot see anything on the skin and asks the nurse what to do. The nurse should make which response? 1."Come to the emergency department." 2."Apply calamine lotion immediately to the exposed skin areas." 3."Take a shower immediately, lathering and rinsing several times." 4."It is not necessary to do anything if you cannot see anything on your skin."

3 When an individual comes in contact with a poison ivy plant, the sap from the plant forms an invisible film on the human skin. The client should be instructed to cleanse the area by showering immediately and to lather the skin several times and rinse each time in running water. Removing the poison ivy sap will decrease the likelihood of irritation. Calamine lotion may be one product recommended for use if dermatitis develops. The client does not need to be seen in the emergency department at this time.

The nurse is caring for a client after the application of a plaster cast for a fractured left radius. The nurse should suspect impairment with the neurovascular status of the client's casted extremity if which findings are noted? Select all that apply. 1.Capillary refill is less than 3 seconds 2.Pulses present and with swollen, pink fingers 3.Client report of severe, deep, unrelenting pain 4.Client report of pain as nurse assesses finger movement 5.Client report of numbness and tingling sensation in the fingers

3, 4, 5 The pressure in compartment syndrome, if unrelieved, will cause permanent damage to nerve and muscle tissue distal to the pressure. Circulatory damage may result in necrosis. Nerve and muscle damage may result in permanent contractures, deformity of the extremity, and functional impairment. Normal capillary refill time is 3 seconds or less. Pink appearance and a pulse indicate adequate blood flow; swelling is expected after a fracture. Client report of severe, deep, unrelenting pain; client report of numbness and tingling sensation; and client report of pain as the nurse assesses finger movement are indicative of development of compartment syndrome.

The nurse notes that an older adult has a number of bright, ruby-colored, round lesions scattered on the trunk and thighs. How should the nurse document these lesions in the medical record? 1.Venous stars noted on trunk and thighs 2.Spider angiomas observed on trunk and thighs 3.Appears to have purpura on trunk and thighs 4.Appears to have cherry angiomas on trunk and thighs

4 A cherry angioma occurs with increasing age and has no clinical significance. It is noted by the appearance of small, bright, ruby-colored round lesions on the trunk and/or extremities. A venous star results from increased pressure in veins, usually in the lower legs, and has an irregularly shaped bluish center with radiating branches. Spider angiomas have a bright red center, with legs that radiate outward. These are commonly seen in those with liver disease or vitamin B deficiency, although they can occur occasionally without underlying pathology. Purpura results from hemorrhage into the skin.

The nurse is caring for a client with a diagnosis of Cushing's syndrome. Which expected signs and symptoms should the nurse monitor for? Select all that apply. 1.Anorexia 2.Dizziness 3.Weight loss 4.Moon face 5.Hypertension 6.Truncal obesity

4, 5, 6 A client with Cushing's syndrome may exhibit a number of different manifestations. These could include moon face, truncal obesity, and a buffalo hump fat pad. Other signs include hypokalemia, peripheral edema, hypertension, increased appetite, and weight gain. Dizziness is not part of the clinical picture for this disorder.

The nurse has suggested specific leg exercises for a client immobilized in right skeletal lower leg traction. The nurse determines that the client needs further instruction if the nurse observes the client performing which action? 1.Pulling up using the trapeze 2.Flexing and extending the feet 3.Doing quadriceps-setting and gluteal-setting exercises 4.Performing active range of motion to the right ankle and knee

4 Active range of motion to the right ankle and knee would disrupt skeletal traction of the right lower leg. The client may pull up using the trapeze, perform active range of motion with uninvolved joints, and do isometric muscle-setting exercises (such as quadriceps- and gluteal-setting exercises). The client also may flex and extend the feet. These exercises are within therapeutic limits for the client in skeletal traction to maintain muscle strength and range of motion.

A client with long-standing empyema undergoes decortication of the affected lung area. Postoperatively the nurse should place the client in which position? 1.Sims' 2.Supine 3.Side-lying 4.Semi Fowler's

4 After any procedure involving lung surgery, the nurse should position the client in semi Fowler's position. This position allows for maximal lung expansion and promotes drainage through the chest tube that may be placed during surgery. The positions identified in the remaining options will limit lung expansion.

During routine nursing assessment after hypophysectomy, a client complains of thirst and frequent urination. Knowing the expected complications of this surgery, what should the nurse assess next? 1.Serum glucose 2.Blood pressure 3.Respiratory rate 4.Urine specific gravity

4 After hypophysectomy, temporary diabetes insipidus can result from antidiuretic hormone deficiency. This deficiency is related to surgical manipulation. The nurse should assess urine specific gravity and notify the health care provider if the result is less than 1.005. Although the remaining options may be components of the assessment, the nurse would next assess urine specific gravity.

A client arriving at the emergency department has experienced frostbite to the right hand. Which finding would the nurse note on assessment of the client's hand? 1.A pink, edematous hand 2.Fiery red skin with edema in the nail beds 3.Black fingertips surrounded by an erythematous rash 4.A white color to the skin, which is insensitive to touch

4 Assessment findings in frostbite include a white or blue color; the skin will be hard, cold, and insensitive to touch. As thawing occurs, flushing of the skin, the development of blisters or blebs, or tissue edema appears. Options 1, 2, and 3 are incorrect.

A client complains of chronic pruritus. Which diagnosis should the nurse expect to note documented in the client's medical record that would support this client's complaint? 1.Anemia 2.Hypothyroidism 3.Diabetes mellitus 4.Chronic kidney disease

4 Clients with chronic kidney disease often have pruritus, or itchy skin. This is because of impaired clearance of waste products by the kidneys. The client who is markedly anemic is likely to have pale skin. Hypothyroidism may lead to complaints of dry skin. Clients with diabetes mellitus are at risk for skin infections and skin breakdown.

A client is admitted with suspected diabetic ketoacidosis (DKA). Which clinical manifestations best support a diagnosis of DKA? 1.Blood glucose 500 mg/dL (27.8 mmol/L); arterial blood gases: pH 7.30, PaCo2 50, HCO3- 26. 2.Blood glucose 400 mg/dL (22.2 mmol/L); arterial blood gases: pH 7.38, PaCo2 40, HCO3- 22. 3.Blood glucose 450 mg/dL (25.0 mmol/L); arterial blood gases: pH 7.48, PaCo2 39, HCO3- 29. 4.Blood glucose 350 mg/dL (19.4 mmol/L); arterial blood gases: pH 7.28, PaCo2 30, HCO3- 14.

4 DKA is caused by a profound deficiency of insulin and is characterized by hyperglycemia (blood glucose level greater than or equal to 250 mg/dL [13.9 mmol/L]), ketosis (ketones in urine or serum), metabolic acidosis, and dehydration. The correct option is 4, as it represents an elevated blood glucose and the arterial blood gases (ABGs) indicate metabolic acidosis. Option 1 is incorrect, as the ABGs indicate respiratory acidosis; option 2 is incorrect, as the ABG values are within normal; and option 3 is incorrect, as the ABGs indicate metabolic alkalosis.

The nurse is providing education to a client with type 2 diabetes mellitus. The nurse explains in layperson's language the physiological mechanism behind hypoglycemia. Which response by the client determines that teaching has been successful? 1. "My body cannot make insulin." 2."My body has decreased epinephrine levels." 3."My body decreases release of cortisol, which is a stress hormone." 4."My body increases glucagon production to fight low blood sugars."

4 Glucagon is secreted from the alpha cells in the pancreas in response to declining blood glucose levels. At the same time, hypoglycemia triggers increased cortisol release, increased epinephrine release, and decreased secretion of insulin. Options 1, 2, and 3 are not physiological mechanisms that take place to combat the decrease in the blood glucose level.

A young man with type 1 diabetes mellitus tells the nurse that he might lose his job because he has been having frequent hypoglycemic reactions. His boss thinks that he is drunk during these episodes and that he has been drinking on the job. Which action by the nurse would best assist this client to meet his needs? 1.Ask the client if he indeed has been drinking at work. 2.Ask the client what he does to treat his hypoglycemia. 3.Contact the local employment office to help him find another job. 4.Examine factors with the client that may be causing frequent hypoglycemic episodes.

4 Hypoglycemic reactions present as adrenergic symptoms of tremor, shakiness, and nervousness that are comparable or alike to the signs of alcohol intoxication. The best strategy to assist the client to meet his needs is to decrease the episodes of hypoglycemia by first identifying and then eliminating those factors that precipitate this event. Asking the client if he has been drinking at work and contacting the local employment office are inappropriate. Asking the client what he does to treat his hypoglycemia is not directly related to the subject, factors that may cause frequent hypoglycemic episodes.

The nurse is administering a dose of pirbuterol to a client. The nurse should monitor for which side or adverse effect of this medication? 1.Drowsiness 2.Hypokalemia 3.Hyperglycemia 4.Increased pulse

4 Pirbuterol is an adrenergic bronchodilator. Side and adverse effects include tachycardia, hypertension, chest pain, dysrhythmias, nervousness, restlessness, and headache. The nurse monitors for these effects during therapy. The other options are not side and adverse effects of this medication.

The nurse is reviewing the discharge instructions for the client who had a skin biopsy. Which statement, if made by the client, would indicate a need for further instruction? 1."I will keep the dressing dry." 2."I will watch for any drainage from the wound." 3."I will use the antibiotic ointment as prescribed." 4."I will return tomorrow to have the sutures removed."

4 Sutures usually are removed 7 to 10 days after a skin biopsy, depending on health care provider (HCP) preference. After a skin biopsy, the nurse instructs the client to keep the dressing dry and in place for a minimum of 8 hours as prescribed. After the dressing is removed, the site is cleaned once a day with tap water or saline to remove any dry blood or crusts. The HCP may prescribe an antibiotic ointment to minimize local bacterial colonization. The nurse instructs the client to report any redness or excessive drainage at the site. The site may be closed with sutures or may be allowed to heal without suturing.

A young adult client has never had a chest x-ray before and expresses to the nurse a fear of experiencing some form of harm from the test. Which statement by the nurse provides valid reassurance to the client? 1."You'll wear a lead shield to partially protect your organs from harm." 2."The amount of x-ray exposure is not sufficient to cause DNA damage." 3."The test isn't harmful at all. The most frustrating part is the long wait in radiology." 4."The x-ray exam itself is painless, and a lead shield protects you from the minimal radiation."

4 Clients should be taught that the amount of exposure to radiation is minimal and that the test itself is painless. The wording in each of the other options is only partly true and therefore cannot provide valid reassurance to the client.

A client has been diagnosed with gout, and the nurse provides dietary instructions. The nurse determines that the client needs additional teaching if the client states that it is acceptable to eat which food? 1.Carrots 2.Tapioca 3.Chocolate 4.Chicken liver

4 Liver and other organ meats should be omitted from the diet of a client who has gout because of their high purine content. Purines are a form of protein. The food items identified in the other options contain negligible amounts of purines and may be consumed freely by the client with gout.

A client is to receive a prescription for methocarbamol. The nurse provides instructions to the client about the medication. Which client statement would indicate a need for further education? 1."My urine may turn brown or green." 2."I might get some nasal congestion from this medication." 3."This medication is prescribed to help relieve my muscle spasms." 4."If my vision becomes blurred, I don't need to be concerned about it."

4 Methocarbamol is a muscle relaxant that works by blocking nerve impulses (or pain sensations) that are sent to the brain. The client needs to be told that the urine may turn brown, black, or green. Other adverse effects include blurred vision, nasal congestion, urticaria, and rash. The client needs to be instructed to notify the health care provider if these side/adverse effects occur.

The nurse instructs a client to use the pursed-lip method of breathing and evaluates the teaching by asking the client about the purpose of this type of breathing. The nurse determines that the client understands if the client states that the primary purpose of pursed-lip breathing is to promote which outcome? 1.Promote oxygen intake. 2.Strengthen the diaphragm. 3.Strengthen the intercostal muscles. 4.Promote carbon dioxide elimination.

4 Pursed-lip breathing facilitates maximal expiration for clients with obstructive lung disease. This type of breathing allows better expiration by increasing airway pressure that keeps air passages open during exhalation. Options 1, 2, and 3 are not the purposes of this type of breathing.

The nurse is caring for a client who was admitted to the burn unit after sustaining a burn injury covering 30% of the body. What is the most appropriate time frame for the emergent phase? 1.The entire period of time during which rehabilitation occurs 2.The period from the time the client is stable to the time when all burns are covered with skin 3.The period from the time the burn was incurred to the time when the client is admitted to the hospital 4.The period from the time the burn was incurred to the time when the client is considered physiologically stable

4 The emergent phase of burn care generally extends from the time the burn injury is incurred until the time when the client is considered physiologically stable. The acute phase lasts until all full-thickness burns are covered with skin. The rehabilitation period lasts approximately 5 years for an adult and includes reintegration into society.

An older client is lying in a supine position. The nurse understands that the client is at least risk for skin breakdown in which body area? 1.Heels 2.Sacrum 3.Back of the head 4.Greater trochanter

4 The greater trochanter is at greater risk of skin breakdown from excessive pressure when the client is in the side-lying position. When the client is lying supine, the heels, sacrum, and back of the head all are at risk, as are the elbows and scapulae.

The nurse is caring for a hospitalized client who is retaining carbon dioxide (CO2) because of respiratory disease. The nurse anticipates which physical response will initially occur? 1.The client will lose consciousness. 2.The client's sodium and chloride levels will rise. 3.The client will complain of facial numbness and tingling. 4.The client's arterial blood gas results will reflect acidosis.

4 When the client with respiratory disease retains CO2, a rise in CO2 will occur. This results in a corresponding fall in pH, thus respiratory acidosis. This concept forms the basis for key aspects of acid-base balance. The other options are incorrect and are not associated with this initial physical response.

When assessing a lesion diagnosed as basal cell carcinoma, the nurse most likely expects to note which findings? Select all that apply. 1.An irregularly shaped lesion 2.A small papule with a dry, rough scale 3.A firm, nodular lesion topped with crust 4.A pearly papule with a central crater and a waxy border 5.Location in the bald spot atop the head that is exposed to outdoor sunlight

4, 5 Basal cell carcinoma appears as a pearly papule with a central crater and rolled waxy border. Exposure to ultraviolet sunlight is a major risk factor. A melanoma is an irregularly shaped pigmented papule or plaque with a red-, white-, or blue-toned color. Actinic keratosis, a premalignant lesion, appears as a small macule or papule with a dry, rough, adherent yellow or brown scale. Squamous cell carcinoma is a firm, nodular lesion topped with a crust or a central area of ulceration.


Conjuntos de estudio relacionados

Rise of Humans through Neolithic Revolution

View Set

Pediatric Chronic Diarrhea in Children

View Set

CH.10 Incremental Analysis: The Key to Decision-Making

View Set

Ch 14 and 15 public speaking exam

View Set

ch.2 collecting subjective data; interview & health history

View Set